You are on page 1of 105

Ain Shams University

Faculty of Education
Department of Mathematics

Statics 2
(Vector Analysis)

Second Year
(First Semester)
Contents
subject page
Chapter 1: (vector Analysis)
1 Vector algebra 1
2 Exercises 6
3 Vector differentiation 7
4 Curvilinear coordinates 10
5 The differentiation of unit vectors 16
6 Vector Integration 19
7 Exercises 32
Chapter 2: (Vector Operators)
1 Gradient of Scalar Function 35
2 Exercises 39
3 Vector Field Divergence 40
4 Exercises 44
5 The Curl of a Vector Field 46
6 Exercises 58
Chapter 3: (Field and Potential Theory)
1 Conservatives Fields 61
2 Exercises 64
3 Solid Angle 65
4 Attraction and Potential 66
5 Exercises 83
Chapter 4 : (Hydrostatics)
1 Fluid Statics 85
2 Exercises 92
Chapter 5 : (Moment of Inertia)
1 Fundamental Theorems and Solved Problems 93
2 Exercises 103
Chapter 1

(Vector Analysis)

(Vector Algebra) :
A Scalar :- is a quantity having magnitude but no direction, e.g. mass, length, time,
temperature ,and any real number. Operations with scalars follow the same rules as in
elementary algebra.

A Vector:- is a quantity having both magnitude and direction, such as displacement,


velocity, force, and acceleration.


Analytically a vector is represented by a letter with an arrow over it, as A or A or A

And its magnitude is denoted by | A | or | A | or | A | or A .

Some important rules of vectors:


(1) If we have two vectors A , B which their components are A = (A x , A y , A z ) ,
B = (B x , B y , B z ) , then they are equal (iff) ,
A=B ⇔ A x = Bx , A y = By , A z = Bz
(2) The null or zero vector is of the form
0 = (0, 0, 0)
(3) The unit vector: - is a vector having unit magnitude and symbolized as e or  ,
A
Aˆ =
A
(4) The rectangular unit vectors {i, j , k }: they are unit vectors in the direction of
X,Y,Z respectively, such as i = (1,0,0), j = (0,1,0), k = (0,0,1) .

(5) Direction cosines: suppose that the angles made by (r ) OP with the coordinates
OX , OY, OZ are α , β, γ respectively, then l = cos α , m = cos β , n = cos γ are
called
the direction cosines of (r ) OP .

Suppose that the point N is the intersection point from P with OX from the triangle
OPN, then we find that

x = r cos α = r l (1)

if the angle α is acute then the coordinate x and the value cos α are positive,and
if it is obtuse then the coordinate x and the value cos α are negative.
1
similarly

y = r cos β = rm , z = r cos γ = rn (2)

then from (1),(2)


x y z
= , m= , n=
r r r
but
r 2 = x2 + y2 + z2
then
2 + m2 + n2 =1

(6) Laws of vector algebra.

(i) A + B = B + A (commutative law for addition)


(ii) (A + B) + C = A+(B + C) (Associative law for addition)
(iii) (m + n) A = mA + nA (Distributive Law)
(iv) m(A + B) = mA + mB ( Distributive Law)
(v) n(mA) = (nm)A = nmA (Associative law for multiplication)
(vi) mA = Am (commutative law for multiplication)

(7) Multiplication of vectors.


a- The dot or scalar product of two vectors A , B is defined as the product of the
magnitude of A and B and the cosine of the angle θ between them. In symbols,

A . B = A B cos θ = A B cos θ

The following laws are valid


(1) A.B = B.A
(2) i.i = j. j = k .k = 1 , i. j = j.k = k .i = 0
(3) If A = (A x , A y , A z ) , B = (B x , B y , B z )

Then A.B = A x B x + A y B y + A z B z

(4) If A.B = 0 . and A , B are not null vectors, then A and B are perpendicular.
U U U U

b- The cross or vector product of two vectors A , B is defined as

A∧B = A B sin θ e = A B sin θ e .

Which e is the normal unit vector on the plane which contains A , B

2
The physical meaning of cross product between two vectors A , B is the area of a
parallelogram with sides A , B .

The following laws are valid


(1) A ∧ B = B ∧ A
(2) i ∧ .i = j ∧ . j = k ∧ k = 0
i ∧ j = k, j ∧ k = i, k ∧i = j
j ∧ i = −k , k ∧ j = −i, i ∧ k = − j
(3) If A = (A x , A y , A z ) , B = (B x , B y , B z )
then
i j k
A ∧ B = Ax Ay Az
Bx By Bz

= (A y B z − A z B y )i + (A z B x − A x B z ) j + (A x B y − B y B x )k

c- Triple scalar product:


If we have three vectors A , B , and C
then

Ax Ay Az
A.(B ∧ C) = B x By Bz
Cx Cy Cz

The absolute value of the scalar triple product of the three vectors A , B , and C
represents the volume of the parallelepiped whose edges are A , B , and C.

If A.(B∧C) = 0, then it means that the three vectors A , B , and C.are lieng in the
same plane.

d- The triple vector product:


For three vectors A , B , and C. the triple vector product is

A∧(B∧C) = (A.C)B – (A.B)C or (A∧B) ∧C = (A.C)B – (B.C)A

(8) Equation level in the space

3
If we have three points is not a straighter one passes by the plane be equated to
Next Image

x − x1 y − y1 z − z1
x 2 − x1 y 2 − y1 z 2 − z1 = 0
x 3 − x1 y 3 − y1 z 3 − z1

Thus, we get the equation plane, taking into account the three points are

A( x 1 , y 1 , z 1 ), B(z 2 , y 2 , z 2 ), C( x 3 , y 2 , z 3 )

The point D(x, y, z) is the general point on the plane .

‫إﻟﻐﺎء‬
It can find Equation plane another way using point in the level and vector
perpendicular to the plane.
If we have the point P0 ( x 0 , y 0 , z 0 ) which its position vector is r 0 attributed to the point
of origin. n is the normal vector on the plane and P(x, y, z) is a general point in the plane
and its position vector is r. To be the point P(x, y, z) located on the plane must be a
vector between P 0 to P, P0 P = (r − r 0 ) is normal to the vector n.

P0 P.n = 0 (perpendicular condition)


(r − r 0 ).n = 0 (the cartisian form of the plane equation)
⇒ λ ( x − x 0 ) + µ( y − y 0 ) + ν ( z − z 0 ) = 0 : n = ( λ , µ, ν )

⇒ λx + µy + νz = d : d = λx 0 + µy 0 + νz 0

The equation of the plane is λ x + µy + νz = 0 and if (d≠0) the equation will be

x y z d d d
+ + = 1: a= , b= , c=
a b c λ µ ν
Which a, b, c represent the parts of principle axes X, Y, Z respectively.

Example:
Find the equation of the normal plane on the vector n = 2i + 3 j + 6k which passing
through the point P 0 which its position vector r 0 = i + 5 j + 3k
Solution:
Let the point P is any point on the plane which its position vector is r = x i + y j + z k
Then the equation of the plane is (r − r 0 ).n = 0

4
[ ][ ]
⇒ ( x − 1)i + ( y − 5) j + (z − 3)k . 2i + 3 j + 6k = 0

⇒ 2( x − 1) + 3( y − 5) + 6(z − 3) = 0

⇒ 2 + 3y + 6z = 35

Another solution:
the equation of the plane is λx + µy + νz = d, but n = (λ, µ, ν) = (2, 3, 6)

∴2x + 3y + 6z = d
but the plane passing through the point P 0 i.e. the point (1, 5, 3).

∴which achieve the equation of the plane


2 + 15 + 18 d ⇒ d = 35

2x + 3y + 6z = 35

Exercises
(1) If A = 2i − j + k , B = −2i + j − 3k , C = i − 3 j − 2k , D = 3i + 2 j + 5k

5
find the value of α , β , γ such that D =α A+ β B+γ C

(2) find the angle between l A = 2i + 2 j − k & B = 6i − 3 j + 2k

(3) find the value of the parameter λ if the following vectors are perpendicular.
A = λ i + 2 j + k & B = 4i − 2 j − 2k
(4) find the projection of the vector A over a vector B if
A = i − 2 j + k & B = 4i − 4 j + 7 k
(5) prove by using vectors that the triangle with vertices at the point
(10,−1,6) & (2,4,3) & (4,1,9) Is right angle triangle and equilateral triangle.
(6) prove that the following vectors are located in one level.
A = i + 2 j − 2k , B = 2i + j + 2k , C = 4i − j + 10k

(i ) ( A . B ) + ( A . ∧ B ) = A 2 B 2
2 2
(7) Prove that
(ii ) A ∧ (B ∧ C ) + B ∧ (C ∧ A) + C ∧ ( A ∧ B ) = 0
(8) if A , B are the position vectors of the points P & Q respectively, find the equation of
the plane pass through the point Q and normal to the line PQ ,where
A = 3i + j + 2k & B = i − 2 j − 4k
(9) find the equation of the plane which pass through the points
P1 ( x1 , y1 , z1 ) & P2 ( x 2 , y 2 , z 2 ) & P3 ( x3 , y 3 , z 3 )
(10) find the equation of the plane which pass through the point P(− 1,−4,6 ) and the vector
A = i + 2 j − 2k is perpendicular to it. Find also the cartisian and parametric
Equations for normal line on this plane and pass through the point P.
(11) find the equation of the straight line which pass through the points (0,2,3) & (1,1,1)
and the equation of the normal plane on it and pass through the point (0,0,−2 ) and
thenfind their intersection point.
(12) if x . a = 0 & b − x = λ a , prove that
. x=
[(a ∧ b ) ∧ a ] & λ = a . b
a2 a2
(13) if λ ≠ 0 ,prove that the solution of the equation λ x + a ∧ x = b is of the form
λ2 b + (a . b )a + λ (b ∧ a )
x=
λ (a 2 + b 2 )
(14) find the unit vectors in the directions of a + b & a − b ,where
a = 2e1 + 3e 2 , b = 5e1 + 12e 2 , where e1 & e 2 are the unit vectors in the direction
which the angle 60 ⊗ between them.

Vector Differentiation

Ordinary Derivatives of Vectors


Suppose that the vector R(u) is a function of the scalar variable u, then

6
∆ R R(u + ∆u ) − R(u )
=
∆u ∆u
Where ∆u denotes an increment in u.
The ordinary derivative of the vector R
dR R (u + ∆u ) − R (u )
= lim ,
du ∆u →0 ∆u
dR
We note that is a vector depending on u
du
d2R d3R
If this derivative exists then we can find , ,....
du 2 du 3
Space Curves : If R(u) is the position vector joining the origin O of a coordinate system
And any point (x, y, z), then

R (u ) = r (u )
= x (u )i + y(u ) j + z(u )k
As u changes, the terminal point of r describes a space curve having parametric equations
x = x(u) , y = y(u) , z = z(u)

then
dr r (u + ∆u ) − r (u )
= lim
du ∆u →0 ∆u

Is a curve in the direction of the tangent to the space curve at (x, y, z) and is given by
d r dx dy dz
= i+ j+ k
du du du du
dr
If u is the time, represents the velocity v which in the direction of the tangent.
dt
dv d 2 r
Similarly, = represents its acceleration along the curve.
dt dt 2

Differentiation Formulae:

d d A dB
(i ) ( A ± B) = ±
du du du
d dB d A
(ii ) ( A.B) = A. + .B
du du du
d dB d A
(iii ) ( A ∧ B) = A ∧ + ∧B
du du du

7
d d A dφ
(iv ) (φ A) = φ + A
du du du
d  dC   dB  dA
(v) ( A.( B ∧ C )) = A. B ∧  + A. ∧C + .( B ∧ C )
du  du   du  du
d  dC   dB  dA
(vi ) ( A ∧ ( B ∧ C )) = A ∧  B ∧ + A∧ ∧C + ∧ (B ∧ C)
du  du   du  du

Partial Derivatives of Vectors

If A is a vector depending on more than one scalar variable, say (x, y, z) for example

Then we can write A = A(x, y, z). The partial derivatives of A with respect to x
Is defined as

∂A A( x + ∆x , y, z) − A( x , y, z)
= lim
∂x ∆x →0 ∆x

∂A ∂A
If this limit exists, similarly , are the partial derivatives of A with respect to y
∂z ∂y
and z respectively.
Higher derivatives can be defined as in the calculus.

∂ 2 A ∂  ∂A  ∂ 2 A ∂  ∂A  ∂ 2 A ∂  ∂A 
=  , =  , =  
∂x 2 ∂x  ∂x  ∂y 2 ∂y  ∂y  ∂z 2 ∂z  ∂z 

∂2 A ∂  ∂A  ∂2 A ∂  ∂A  ∂3 A ∂  ∂A 
=  , =  , =  
∂x∂y ∂x  ∂y  ∂y∂x ∂y  ∂x  ∂x∂z 2
∂x  ∂z 2 
∂2 A ∂2 A
we note that =
∂x∂y ∂y∂x

Rules for partial differentiation of vectors are similar to those used in elementary calculus
for scalar functions. Thus if B , A are functions of (x, y, z) then,

∂ ∂A ∂A
(1) (A.B) = A. + .B
∂x ∂x ∂x
∂ ∂A ∂A
(2) (A ∧ B) = A ∧ + ∧B
∂x ∂x ∂x

8
∂2 ∂ ∂  ∂ ∂ ∂B 
(3) (A.B) = ( A.B) = .B + A.
∂y∂x ∂y  ∂x  ∂y  ∂x
  ∂x 

∂ 2 B ∂A ∂B ∂A ∂B ∂ 2 A
= A. + . + . + .B
∂y∂x ∂x ∂y ∂y ∂x ∂y∂x

Differentials of vectors:

(1) if A = A1 i + A2 j + A3 k
then d A = dA1 i + dA2 j + dA3 k
(2) d ( A.B) = d A.B + A.d B
(3) d ( A ∧ B) = d A ∧ B + A ∧ d B
(4) if A = A( x , y, z)
then
∂A ∂A ∂A
dA= dx + dy + dz
∂x ∂y ∂z
 ∂A ∂A ∂A 
=  i+ j+ (
k . dxi + dy j + dz k
∂z 
)
 ∂x ∂y
= ∇ A.d r
Where the operator ∇ is called nabla and is defined as

∂ ∂ ∂
∇=i + j +k
∂x ∂y ∂z
and r = xi + y j + zk then d r = dx i + dy j + dz k

Examples:

dA dA
(1) If A has constant magnitude, show that A, are perpendicular provided ≠0
dt dt
Solution:
| A |= A 2 = A.A
Then,
dA dA
0 = A. + .A
dt dt
dA dA
∴ 2 A. =0 ⇒ A. =0
dt dt
i.e. the two vectors are perpendicular.

dA dA
(2) Prove that: A, =A
dt dt
The solution:
 A 2 = A.A ,

9
𝒅 𝒅
(𝑨. 𝑨) = �𝑨𝟐 �.
𝒅𝒕 𝒅𝒕

𝒅 𝒅𝑨 𝒅𝑨 𝒅𝑨 𝒅 𝒅𝑨
�𝑨. 𝑨) = 𝑨. + . 𝑨 = 𝟐𝑨. 𝒂𝒏𝒅 �𝑨𝟐 � = 𝟐 𝑨 �
𝒅𝒕 𝒅𝒕 𝒅𝒕 𝒅𝒕 𝒅𝒕 𝒅𝒕

𝑑𝐴 𝑑𝐴 𝑑𝐴 𝑑𝐴
Then 2 𝐴. = 2𝐴 or 𝐴. =𝐴
𝑑𝑡 𝑑𝑡 𝑑𝑡 𝑑𝑡

(3) A particle moves along the curve x = 2 t 2 , y = t 2 − 4 t , z = 3t − 5 where t is the


time. Find the components of its velocity and acceleration at time t = 1 in the
direction i − 3 j + 2k .
:‫اﻟحﻞ‬

(i) the velocity ( v) =


dr d
=
dt dt
[
2 t 2 i + ( t 2 − 4 t ) j + (3t − 5)k ]
= 4 t i + (2 t − 4) j + 3k
v t =1 = 4i − 2 j + 3k
unit vector in the direction i − 3 j + 2k = (i − 3 j + 2k ) / 14 the component of the
velocity in the velocity in the given direction is
i − 3 j + 2k 16 8
(4i − 2 j + 3k ) = = 14
14 14 7
dv d 2 r
(ii) the acceleration (a) = = 2 = 4i + 2 j + 0k = 4i + 2 j
dt dt
a t =1 = 4i + 2 j
the component of the acceleration in the given direction is
i − 3 j + 2k − 2 14
(4i + 2 j). = =− .
14 14 7

Curvilinear coordinates

Let the rectangular coordinates (x, y, z) of any point P be expressed as functions of


u1 , u2 , u3

x = x (u 1 , u 2 , u 3 ) 

y = y( u 1 , u 2 , u 3 )  (1)

z = z(u 1 , u 2 , u 3 ) 

Suppose that (1) can be solved for u 1 ، u 2 ، u 3 R R in terms of x, y, z, i.e. R

10
u 1 = u 1 ( x , y, z ) 

u 2 = u 2 ( x , y, z )  (2)

u 3 = u 3 ( x , y, z) 

Given a point P with rectangular coordinates (x,y,z), we can from (2) associate a
Unique set of coordinates u 1 , u 2 , u 3 called the curvilinear coordinates of P.

u 1 ( x , y, z ) = c 1 , 

u 2 ( x , y, z ) = c 2 ,  (3)

u 3 ( x , y, z) = c 3 

The surfaces (3) are called coordinate surfaces, and each pair of these surfaces intersect
in curves called coordinate curves.
If these coordinate curves intersect at right angles, the curvilinear coordinate system
Is called orthogonal.

Unit vectors in curvilinear systems


Let r = xi + y j + z k be a position vector of a point P. Then (1) can be written as
r = r (u1 , u 2 , u3 ) (5)
If e 1 , e 2 , e 3 are the position vectors in the direction of the tangent for curves
∂r ∂r ∂r ∂r
(u1 , u 2 , u 3 ) respectively and d r = du 1 + du 2 + 3 du 3 , then is the tangent
∂u 1 ∂u 2 ∂u 3 ∂u 1
∂r ∂r
vector for the curve u 1 at the point P, similarly , are the tangent vectors for
∂u 3 ∂u 2
the curves u 3 , u 2 respectively.
∂r
Then the unit vector e 1 in the direction of is
∂u 1

∂r ∂r ∂r
∂u 1 ∂u 2 ∂u 3
e1 = (6) e2 = (7) e3 = (8)
∂r ∂r ∂r
∂u 1 ∂u 2 ∂u 3

Equations (6),(7) and (8) give the unit vectors in the directions of increasing u 1 , u 2 , u 3
respectively.

11
∂r ∂r ∂r
h1 = , h2 = , h3 =
∂u 1 ∂u 2 ∂u 3

h 1 , h 2 , h 3 Are called (scale factors), then

1 ∂r 1 ∂r 1 ∂r
e1 = , e2 = , e3 = (10)
h 1 ∂u 1 h 2 ∂u 2 h 3 ∂u 3

Arc length, Area and volume elements:

Where r = r (u 1 , u 2 , u 3 )
Then
∂r ∂r ∂r
dr = du 1 + du 2 + du 3
∂u 1 ∂u 2 ∂u 3

= h 1 du 1 e 1 + h 2 du 2 e 2 + h 3 du 3 e 3

We can find the arc length from the relation

We notice that on the curve u 1 , the curves u 2 , u 3 are constants, i.e. d r = h 1 du 1 e 1 . i.e.
the
arc length d 1 in the direction of u 1 at the point P is d 1 = h 1 du 1 .
The general form for arc length is

d n = h n du n , n = 1, 2, 3, …

The area element

ds 1 = (h 2 du 2 e 2 ) ∧ (h 3 du 3 e 3 ) = h 2 h 3 du 2 du 3
similarly

ds 2 = h 1 h 3 du 1 du 3 , ds 3 = h 1 h 2 du 1 du 2
Where ds 1 is the area element on the tangent for the curve u 1 and so on
And the volume element is

dv = h 1 h 2 h 3 du 1 du 2 du 3

Special cases:

(i) The Cartesian coordinates:

12
u 1 = x, u2 = y , u3 = z
the curves u 1 , u 2 , u 3 parallel the coordinates X, Y , Z respectively.

Then the arc length is

(d) 2 = (dx ) 2 + (dy) 2 + (dz) 2


i.e. h 1 = h 2 = h 3 = 1
the area length is
ds 3 = dx dy , ds 2 = dx dz , ds 1 = dy dz

the volume element is dv = dx dy dz

(b) The cylindrical coordinates:

u 1 = r, u 2 = φ , u3 = z

conversion equations from Cartesian to cylindrical coordinates:

r= x2 + y 2 x = r cos φ 


φ = tan −1 
y
  ⇐ y = r sin φ 
 x 
z=z z=z 

Where the position vector of the general point is of he form:

r = xi + y j + z k
= r cos φ i + r sin φ j + z k

∂r
h1 = = cos φ i + sin φ j = cos 2 φ + sin 2 φ = 1
∂r

∂r
Then h2 = = − r cos φ i + r sin φ j = r
∂φ

∂r
h3 = =1
∂z

We note that 0 < ρ < ∞ , 0 ≤φ ≤ 2π , − ∞ < z < ∞

13
It is easy to get the unit vectors in the directions of ρ , φ , z by using the unit vectors in
the directions of x, y, z . For example

e r = cos φ i + sin φ j 


e φ = − sin φ i + cos φ j I

ez = k 

We can write d r as

d r = dr e r + rdφ e φ + dz e z

From the previous relations

eρ .eφ = eφ .ez = ez .er = 0


Which prove that the cylindrical coordinates are orthogonal.

The arc length is of the form (dl )2 = (dρ )2 + (ρdφ )2 + (dz )2


The area element is ds1 = ρdρdφ , ds 2 = dφdz , ds3 = ρdρdz
The volume element is dv = ρdρdφdz

(c) The spherical coordinates:


In this coordinates u 1 = r , u 2 = θ , u 3 = φ , and 0 ≤ r ≤ ∞ , 0 ≤ θ ≤ π , 0 ≤ φ ≤ 2π
Conversion equations from the Cartesian to spherical coordinates are:

x = r sin θ cos φ

y = r sin θ sin φ
z = r cos θ
And the reversal coordinates are:

 x 2 + y2   y
r = x + y + z , θ = tan 
2 2 2 −1 , φ = tan −1  
 z  x
 
Where the position vector of the point p is of the form

r = xi + y j + zk

14
= r sin θ cos φi + r sin θ sin φ j + r cos θk
and

∂r ∂r ∂r
h1 = =1 , h2 = =r , h3 = = r sin θ
∂r ∂θ ∂φ

d r = dr e r + rdθe θ + r sin θdφe φ

where

e r = sin θ cos φ i + sin θ sin φ j + cos θ k ,


eθ = cos θ cos φ i + cos θ sin φ j − sin θ k ,
e φ = − sin φ i + cos φ j

And the arc length is


(dl )2 = (dr )2 + r 2 (dθ )2 + r 2 sin 2 θ (dφ )2

The surface elements are


ds1 = r 2 sin θ dθ , ds 2 = r sin θ dφdr , ds 3 = rdrdθ
The volume element

dv = r 2 sin θ drdθdφ

Kind of coordinates h1 h2 h3
Cartesian (x,y,z) 1 1 1
Cylindrical (r, φ,z) 1 r 1
Spherical (r,θ,φ)
1 r r sin θ

15
The differentiation of unit vectors.

If e1 , e 2 , e 3 are unit vectors in the directions of the coordinates u 1 , u 2 , u 3

1 ∂r 1 ∂r 1 ∂r
e1 = , e2 = , e3 =
h1 ∂u1 h2 ∂u 2 h3 ∂u 3

Since

0 ⇒ e1 .e 2 = ∂r ∂r
. = 0 (*)
∂u1 ∂u 2

By differentiating (*) w.r.t u 3 we get

∂r ∂2 r ∂2 r ∂r
. + . = 0 (1)
∂u1 ∂u 2 ∂u3 ∂u1∂u3 ∂u 2

Similarly from the relations


e3 . e1 = 0 , e 2 . e3 = 0

We get

∂r ∂2 r ∂2 r ∂r
. + . = 0 (2)
∂u 2 ∂u1∂u3 ∂u1∂u 2 ∂u3
∂r ∂2 r ∂2 r ∂r
. + . = 0 (3)
∂u3 ∂u1∂u 2 ∂u 2 ∂u3 ∂u1

Multiply equation (1) by (-1) and add to equation (2) and (3)
∂r ∂2 r
. = 0
∂u 3 ∂u 1 ∂u 2

∂2 r
i.e. the vector is normal to the vector e 3 .
∂u1∂u 2

i.e.
∂2 r
= α e1 + β e 2 (4)
∂u1∂u 2
but
∂r ∂r
= h1 e1 , = h 2 e2
∂u1 ∂u 2

16
From equation (4)
∂2 r ∂
= (h1 e1 ) = ∂ (h2 e 2 ) (5)
∂u1∂u 2 ∂u 2 ∂u1
From (4) , (5)
∂h ∂e
e1 1 + h1 1 = α e1 + β e 2
∂u 2 ∂u 2
∂h ∂e
e 2 2 + h2 2 = α e1 + β e 2
∂u1 ∂u1
Which gives
∂h ∂e ∂h ∂e
α e1 = 1 e1 = h2 2 , β e 2 = 2 e 2 = h1 1
∂u 2 ∂u1 ∂u1 ∂u 2

∂ e1 1 ∂h2
i.e = e2 (6)
∂u 2 h1 ∂u1
∂ e 2 1 ∂h1
= e1 (7)
∂u1 h2 ∂u 2

∂ e1 1 ∂h3 ∂ e 2 1 ∂h3
= e3 (8) , = e3 (9)
∂u 3 h1 ∂u1 ∂u 3 h2 ∂u 2

∂ e 3 1 ∂h1 ∂ e 3 1 ∂h2
= e1 (10) , = e2 (11)
∂u1 h3 ∂u 3 ∂u 2 h3 ∂u 3
In general, we can write
∂ e k 1 ∂h j
= ej k≠ j (I )
∂u j hk ∂u k

We know that
∂ e1 ∂
= (e 2 ∧ e 3 )
∂u1 ∂u1
∂e ∂e
= e2 ∧ 3 + 2 ∧ e3
∂u1 ∂u1

From the equations (7),(10) we get

∂ e1  1 ∂h1   1 ∂h1 
= e 2 ∧  e1  +  e1  ∧ e 3
∂u1  h3 ∂u3   h2 ∂u 2  (12)
1 ∂h2 1 ∂h2
= − e3 − e3
h2 ∂u3 h1 ∂u1
Similarly

17
∂e 2 1 ∂h2 1 ∂h2
= − e3 − e1 (13)
∂u 2 h3 ∂u3 h1 ∂u1
and
∂e3 1 ∂h3 1 ∂h3
=− e1 − e2 (14)
∂u3 h1 ∂u1 h1 ∂u 2
In general
∂e k 1 ∂hk 1 ∂hk
=− e k +1 − e k +2 ( II )
∂u k hk +1 ∂u k +1 hk + 2 ∂u k + 2

Examples:
(1) in cylindrical coordinates (r, φ, z)
where h1 = h ρ = 1, h 2 = h φ = ρ, h 3 = h z = 1
We get
∂ er ∂e 1 ∂h2 ∂
= 1 = e2 = (r ) e φ = e φ
∂φ ∂u 2 h1 ∂u1 ∂r
∂ e r ∂ e1 ∂ e r ∂ e1
= = 0, = =0
∂z ∂u 3 ∂r ∂u1
Similarly
∂ eφ ∂ eφ ∂ eφ
= = 0, = − er
∂r ∂z ∂φ
(2) in spherical coordinates

where
h1 = hr = 1, h2 = hθ = r , h3 = hφ = r sin θ
We get
∂ er ∂e r ∂ er
= 0, = eθ , = sin θ e φ
∂r ∂θ ∂φ
∂ eθ ∂ eθ ∂ eθ
= 0, = − er , = r cos θ e φ
∂r ∂θ ∂φ
∂ eφ ∂ eφ ∂ eφ
= 0, = 0, = − sin θ e r − cos θ eθ
∂r ∂θ ∂φ

18
Vector Integration:

(1) Ordinary integrals of vectors:


If the vector R(u) is a function of one variable u, i.e.

R (u ) = R1 (u )i + R 2 (u ) j + R 3 (u ) k

then
∫ R(u )du = i ∫ R1 (u )du + j ∫ R 2 (u )du + k ∫ R 3 (u )du
Is called indefinite integral of R(u).

d S (u )
If there exist a vector S (u ) such that R(u ) = , then
du
d S (u )
∴ ∫ R(u )du = ∫ du
du = S (u ) + C
Where C is an arbitrary constant vector independent of u.

The definite integral between u=a and u=b can be written as:

∫ R(u )du = [S (u ) + C ] = S (b ) − S (a )
b b
a a

Examples:

(1) If ( )
R (u ) = u − u 2 i + 2u 3 j − 3 k , then find ∫ R(u )du
1
2

Solution:

∫ R(u )du = ∫ [(u − u )i + 2u ]


2 2
2 3
j − 3 k du
1 1

( )
2
2 2
= i ∫ u − u 2 du + j ∫ 2u 3 du − k ∫ 3du
1 1
1
2 2
 u2 u3   u2 
+ c 2  − k (3u + c3 ) |12 = − j +
5 15
= i  − + c1  + j  j −3k
 2 3 1  2 1 6 2

d2r
(2) The equation of motion of a particle of mass m is given by m = F (r ) e r ,
dt 2
where r is the position vector, e r is a unit vector in the direction r , show that

dr
, where C is a constant vector. r ∧ =C
dt

19
Solution:

d2 r
 m 2 = F (r ) e r (1)
dt

Multiply both sides of (1) by (r ∧ ) r , then

 d2 r 
m  r ∧ 2  = F (r ) (r ∧ e r ) , and r , e r have the same direction, then
 dt 

r ∧ er = 0
d2 r
r∧ =0 (2)
dt 2
but
d  dr  d 2r dr dr
r ∧ =r ∧ 2 + ∧
dt  dt  dt dt dt

d dr  d2 r
 r ∧  = r ∧ (3)
dt  dt  dt 2
From (2), (3) we get

d dr 
r ∧  = 0
dt  dt 

By integrate both sides we get


dr
r∧ =C
dt

(2) Line Integral:

The line integral of a vector A(u) on a curve C which its equation is

r (u ) = x (u )i + y (u ) j + z (u ) k , between the points P, Q is of the form

∫ A (u ). d r ,
P
and if

A = A1 i + A 2 j + A 3 k , then the line integral is of the form

20
Q Q

∫ A (u ). d r = ∫ ( A1 dx + A2 dy + A3 dz )
P P
The physical meaning of this integral is the work done by the force F on the curve C

Is ∫ F.d r , and if the integral on the curve C is closed, i.e.


C
p → p , then the integral

Is of the form
∫ A.dr . i.e.
C
∫ A . dr = ∫ A . dr
C p
which is called circulation of A about C.

The vector field is called Conservative field if the integral is independent of the path

And depend on the position P,Q, then the line integral of closed field ia equal zero

∫ A .d r = 0 .
C
There is another form of line integral which is called the vector line integral and is
Q

form
∫ A ∧ dρ . of the
ρ
Examples:

(1) if A = (3 x 2 + 6 y )i −14 yz j + 20 xz 2 k , find the line integral


∫ A .dr on the curve C
C
From the point (0,0,0) to (1,1,1) where C is

(i) x = t , y = t2 , z = t3
(ii) the straight lines from (0,0,0) to (1,0,0) then to (1,1,0) and then to (1,1,1).
(iii) the straight line between (0,0,0) to (1,1,1).

Solution:

r = xi + yi + z k

∫ A. d r = ∫ ( A
C C
1 dx + A 2 dx + A3 dx )

[( )
= ∫ 3 x 3 + 6 y dx −14 yzdy + 20 xz 2 dz ]
C

(i) if x = t , y = t 2 , z = t 3 , then the integral of the curve C from the point (0,0,0) to
(1,1,1) is of the form

21
(1,1,1)
∫ A. d r = ∫
C
( 0, 0, 0 )
A. d r

∫ (9t )( )
1
= 2
i −14t 5 j + 20t 7 k . dt i + 2tdt j + 3t 2 dt k
t =0
1
= ∫ 9t 2 dt − 28t 6 dt + 60t 9 dt
t =0

[
= 3t 3 − 4t 7 + 6t 10 t =0 = 5 ]1

(ii) The straight line from (0,0,0) to (1,0,0) then y = 0, z = 0 i.e. dy = dz = 0

∫ [(3x )
+ 0 dx −14(0 ) + 20 (0 ) = ∫ ] (1)
1 1
2
3 x 2 dx =1
x=0 x =0

and the straight line from (0,0,0)to (1,0,0) then x = 1, z = 0 i.e. dx = dz = 0 but y
change
from 0 to 1, then

∫ A . dr = 0
1
(2)
y=0

And the straight line from (1,1,0) to (1,1,1) then x = 1, y = 0 i.e. dx = dy = 0 but z
Change from 0 to 1 then

∫ ∫
1 1
20
A 3 . dz = 20 xz 2 dz = (3)
z =0 0 3

add (1), (2), (3) we get

∫ A . dz = 1 + 0 + 0 = 1
1

z =0

(iii) on the straight line from (0,0,0) to (1,1,1) and its parametric form is

x −0 y−0 z−0
x =t, y = t, z = t where = = =t
1− 0 1− 0 1− 0
Then
1
∫ A . dr = ∫ (3t + 6t )dt − 14t dt + 20dt dt
2 2 2
t =0
C
1
= ∫ (3t 2 + 6t − 14t 2 + 20t 3 )dt
t =0

22
1 13
= ∫ (20t 3 − 11t 2 + 6t )dt =
t =0 3

(2) If F = 3xyi − y 2 j , find


∫ F.dr ,where C is a curve in the plane xy, y = 2x , from
C
2
P P

(0,0) to (1,2).

Solution:
Where C in xy plane, i.e. z = 0

∫C
F .d r = ∫ (3 xyi − y 2 j )(dxi + dy j )
C

= ∫ 3 xydx − y 2 dy
C

y = 2x2 then dy = 4x dx, and x change from 0 to 1.

∫ ∫
1
7
F.d r = (6 x 3 − 16 x 5 )dx = −
C x =0 6

i.e. the movement in the opposite direction .

(3) Find the work done by the force F.

F = (2 x − y − z )i + ( x + y − z 2 ) j + (3x − 2 y + 4 z )k

To move a body one time around the circle C in the plane xy if its center at the
origin and its radius is 3.

Solution:

z = 0 in the plane xy then

∫ F.dr = ∫ (2x − y)dx + (x + y)dy


C C
By using the parametric coordinates for a circle

x = 3 cosθ dx = −3 sinθ dθ
y = 3 sinθ dy = 3 cosθ dθ
if the body rotate one period around the circle, then θ change from 0 to 2π.

23

∫C
F .d r = ∫
θ =0
(6 cos θ − 3 sin θ )(−3 sin θ ) + 3(cos θ + sin θ )(3 cos θ )dθ

=∫ 9(1 − sin θ cos θ )dθ = 18π
θ =0

(4) The acceleration of a particle at any time t ≥ 0 is given by


dv
a= = 12 cos 2t i − 8 sin 2t j + 16t k
dt
If the velocity v and displacement r are zero at t = 0, find v and r at any time.
 a = 12 cos 2 t i − 8 sin 2 t j + 16 t k
Solution:
Integrating with respect to t, we get:
v = i ∫ 12 cos 2tdt + j ∫ − 8 sin 2tdt + k ∫ 16tdt

= i (6 sin 2t ) + j (4 cos 2t ) + k (8t 2 ) + C 1


Since V = 0 at t = 0 then C 1 = −4 j
V = 6 sin 2t i + 4(cos 2t − 1) j + 8t 2 k
Integrating again, we get
r = i ∫ 6 sin 2tdt + j ∫ 4(cos 2t − 1)dt + k ∫ 8t 2 dt
8
∴ r = −3 cos 2t i + (2 sin 2t − 4t ) j + t 3 k + C 2
3
since r = 0 at t = 0 then C 2 = 3i
8
r = 3(1 − cos 2t )i + 2(sin 2t − 2t ) j + t 3 k .
3

(5) If F = xyi − z j + x 2 k and c is the curve x = t2, y = 2t , z = t3. From t = 0 to t=1,

evaluate the line integrals


∫ F ∧ dr
C
Solution
 x = t 2 , y = 2t and z = t 3 ⇒ r = t 2 i + 2t j + t 3 k
∴ d r = dt (2t i + 2 j + 3t 2 k )
i j k
F ∧ d r = dt 2t 3 − t3 [
t 4 = dt (−3t 5 − 2t 4 )i + (2t 5 − 6t 5 ) j + (4t 3 + 2t 4 )k ]
2t 2 3t 2

∫ C
F ∧ dr = ∫
1

t =0
[− (3t 5
]
+ 2t 4 )i − 4t 5 j + 2(2t 3 + t 4 )k dt
1
 1 2  2 1 1 
= −  t 6 + t 5 i − t 6 j + 2 t 4 + t 5 
 2 5  3 2 5  0
 1 2 2 1 1  9 2 7
= −  + i − j + 2 + k  = − i − j + k
 2 5 3  2 5  10 3 5

24
Third:- The surface integral

If we have a vector field A(r) and there is a surface S, then ∫∫ A.d S


S
is the scalar

Surface integral for a vector A on the surface S , where dS


is the vector surface element on this surface. If N is outward normal unit
vector
on the surface element dS , then dS = N dS .

∫∫ A.dS = ∫∫ A.N dS
S S

and the scalar surface integral


∫∫ A.ds is called Flux of A across the surface S.
S
and the vector surface integral for a vector A on the surface S is

∫∫ A ∧ ds = ∫∫ A ∧ ds
S S
There is another surface integrals like

∫∫ φ ds, ∫∫ φ N ds
S S
where φ is a scalar function.

Note:
The symbol
∫∫
S
is used for closed surface integral.

Examples:
(1) Evaluate the suface integral ∫∫ φ dS , where
S
φ = x 2 − 2 xyz + yz 3 , and S is the

square
which vrtices are A(1, 6,−2), B(4, 6, −2), ), C(4, 6, 1), D(1,6, 1).

Solution:
It is clear that the square is in the plane y=6, and its area is in the intervals
−2 < z < 1and , 1<x<4
and dS = dxdz

25
4 1
∴ ∫∫ φdS = ∫ ∫ ( x 2 − 2 xyz + yz 3 )dxdz
x =1 z = 2
S
1
4  z4 
= ∫  x 2 z − 6 xz 2 + 6  dx
x =1
 4  z = −2
4  45 
= ∫  3 x 2 + 18 x −  dx
x =1
 2 
4
 45 
= x3 + 9x 2 − x = 130.5
 2  1

(2) Evaluate the surface integral ∫∫ φ dS , where φ = xz + 3y − z, S is the closed surface


S

contains an octant of a sphere which radius 2 and its centre is the origin and
three

faces each of them is a quarter of a circle in the planes xy , yz , zx.

The solution:

We will use spherical coordinates :


π π
(1) The octant of the sphere r=2 , 0<θ< , 0<φ<
2 2

x = 2 sin θ cos φ
y = 2 sin θ sin φ, z = 2 cos θ

ds 1 = r 2 sin θdθdφ

= 4 sin θdθdφ

(2) The quarter of the circle in xy


π π
0<r<2 , θ= , 0<φ<
2 2
x = r cosθ , y = r sinθ , z = 0
ds 2 = r sinθ drdθ = rdrdφ

(3) The quarter of the circle in xz


π
0 < r < 2, 0 < θ < , φ=0
2

26
ds 3 = rdrdθ x = r sinθ , y = 0 , z = r cosθ
R R

(4) The quarter of the circle in yz


π π
0 < r < 2, 0 < θ < , φ =
2 2
x = 0 , y = r sinθ , z = r cosθ
ds 4 = rdrdθ
then
π π

∫∫ φdS = ∫ ∫
S1
1
2 2
0 0
(4 sin θ cos θ cos φ + 6 sin θ sin φ − 2 cos θ )4 sin θdθdφ

π π

0
2 [
= 8∫ 2 sin θ cos θ sin φ − 3 sin θ cos φ − sin θ cos θ (φ )
2 2
] 2
0

π
 3 π 
= 8∫ 2 2 sin 2 θ cos θ + 1 − cos θ ) − sin θ cos θ  dθ (1)
0
 2 2 
π
2 3 1  π 2
= 8 sin 3 θ + θ − sin 2θ  − sin 2 θ 
3 2 2  4 0
 2 3π π  16
= 8 + − = + 4π
3 4 4  3
π
2
∫∫ φdS 2 = ∫
S2
2
0 0 ∫ (0 + 3r sin φ − 0)rdrdφ
(2)
π π
2
= ∫ 3r 2 dr ∫ 2 sin φdφ = r 3
0 0
[ ] [− cos φ ]
2
0
2
0 =8
π
2
∫∫φdS 3 = ∫
S3
2
0 0 ∫ (r sin θ .r cos θ − r cos θ )rdrdθ

π π
2 2
=∫ 2
∫ r 3 sin θ cos θdrdθ − ∫ 2
∫ r 2 cos θdrdθ
0 0 0 0
π π
2 2
= ∫ r 3 dr ∫ 2 sin θ cos θdθ − ∫ r 2 dr ∫ 2 cos θdθ
0 0 0 0

2 π 2
 r4  1 2   r3  π
(sin θ ) 02 2
2
=    sin θ  −   =−
 4  0 2  0  3 0
3
π
2
∫∫φ.dS
S4
4 =∫
0 0
2
∫ (3r sin θ − r cos θ )rdrdθ

π
2 16
= ∫ r 2 dr ∫ 2 (r sin θ − cos θ )dθ =
0 0 3
16 16 2
∫∫φdS = + 4π + 8 + − = 4π + 18 .
S
3 3 3

27
(3) Evaluate the surface integral
∫∫ φ ds , where φ = xz + 3y − z, and the surface S
S
contains half cylinder which axes is z and its radius is 3, and rectangular in xz plane
and
two half circles in the planes z = 0 , z = 4.

Soluion:
we will use cylindrical coordinates for all the surface except the rectangular which
we will use cartisian coordinates, then we have

(1) The half cylinder ρ = 3, 0 < φ < π , 0 < z < 4, dS1 = 3dφ dz ,
(2) The under half circle 0 < ρ < 3, 0 < φ < π , z = 0, dS 2 = ρdρdφ .
(3) The upper half circle 0 < ρ < 3 , 0 < φ < π , z = 4, dS 3 = ρdρdφ
(4) The rectangular −3 < x < 3, y = 0, 0 < z < 4, ds 4 = dx dz.
In cylindrical coordinates.
x = ρ cos φ , y = ρ sin φ , z = 2

4 π
∫∫φdS = ∫ ∫
S1
1
0 0
(3 z cos φ + 9 sin φ − z )3dφdz

4 ,π
3 1 
= 3 z 2 sin φ − 9 z cos φ − z 2φ 
2 2  z =0,φ =0
= 3[9(4)(2) − 8π ] = 24(9 − π )
3ρ sin φ .ρdφdρ = [ ρ 3 ]0ρ [− cos φ ]0 = 54
3 π
∫∫ φdS =∫ ∫
π
2
0 0
S2
3 π
∫∫ φdS 3 = ∫
S3

0 0
(4 ρ cos φ + 3ρ sin φ − 4) ρdρdφ

3,π
3 
=  ρ 3 sin φ − ρ 3 cos φ − 2φρ 2 
4  ρ =0,φ =0
= 27(2) − 18π = 54 − 18π
3
 x2  1 
4
4 π
∫∫ φdS 4 = ∫ ∫ ( xz − z )dxdz =  − x   z 2  = −48 .
S4
0 0
2  −3  2  0
∴ ∫∫ φdS = 24(9 − π ) + 54 + 54 − 18π − 48 = 276 − 42π
S

Fourth: The Volume Integral

We consider closed surface S contain a volume V ,then the volume integral for
The vector A over the volume V is defined as ∫∫∫ A dV .
V

Examples;

28
(1) Evaluate the volume integral
∫∫∫ φ dv ,where φ ( x, y, z) = x
V
2
− 2 xyz + yz 3 , V is the

volume of the cylinder which its radius 2 , the height 4 , the axis is z and the base is
in
the xy plane.

Solution:
We use cylindrical coordinates ,where

0 < ρ < 2, 0 < φ < 2π , 0 < z < 4 , dv = ρ dρ dφ dz


2 4 2π
[
∴ ∫∫∫φdv = ∫ ρdρ ∫ dz ∫ dφ ρ cos 2 φ − 2 ρ 2 cos φ sin φz + zρ 3 sin φ
0 0 0
2
]
V

2π 2π
1 1 
∫ cos φdθ = φ + sin 2φ = π
2
0 2 2 0
2π 2π

∫ cos φ sin φdφ = 0,


0 ∫ sin φdφ = 0
0

then

∫∫∫ φdv = 4π.


4
[ ]
1 2
ρ
2
0 = 16π

(2) Evaluate ∫∫∫φdV


V
,where φ = xz + 3y − z , V is the volume contains the closed

surface which contains the octant of the sphere with radius 2 and its center is the
origin and three faces as quarter of a circle in the planes xy , yz , zx.

The solution:
By using spherical coordinates
π π
0 < r < 2, 0 < θ < , 0 < φ < , dV = r 2 sin θdrdθdφ
2 2
x = r sin θ cos φ , y = r sin θ sin φ , z = r cos θ

29
π π
2
∴V = ∫ ∫ ∫
2 2 (r 2 sin θ cosθ cos φ + 3r sin θ sin φ − r cosθ )r 2 dr sin θdθdφ
0 0 0
π
2 π
=∫ ∫2 (r 2 sin 2 θ cosθ + 3r sin 2 θ − r cosθ )r 2 drdθ
0 0 2
π
 1
2 3  1  π 1 2
= ∫ dr r 2 sin 3 θ + r 2 θ − sin 2θ  − r 3 sin 2 θ 
0
 3 2  2  2 2 0
2
12 3 π  r5 π r4  32
= ∫ dr  r 4 + πr 3 − r 3  =  + .  = + 2π
0
3 4 4  15 2 4  0 15

(3) Evaluate ∫∫∫φdV ,where V is the volume in the surface S which contains a half of
V

a cylinder its axis is z , the radius is 3 and rectangular in xz plane and half of two

circles in the planes z = 4 ، z =0. ،φ = xz + 3y − z.

The solution
By using cylindrical coordinates,

0<ρ<3, 0<φ<π, 0<z<4


3 π 4
∫∫∫φdV = ∫ ∫ ∫ ( ρ cos φz + 3ρ sin φ − z ) ρdρdφdz
V
0 0 0

= ∫ [zρ sin φ − 3ρ cos φ − φz ] 0 ρdρdz


3 π

.
0
3, 4
3 π  z2 ρ 2 
=∫ ∫ (6π − zπ ) ρdρdz = 2 zρ 3 − π 
0
 2 2  0, 0
= 216 − 36π

∫∫∫ FdV , where F(x, y, z) = 45x y, and V is the region bounded by


2
(4) Evaluate
V

x = 0 , y = 0 , z = 0, 4x + 2y + z = 8
The solution:

30
2 4 8− 4 x − 2 y
∫∫∫ FdV = ∫ ∫ ∫
V
x =0 y =0 z =0
[45 x 2 y ]dzdydx

= 45∫
2
 4 x 2 y (8 − 4 x − 2 y )dy  dx
 ∫ y =0
x =0   .
2 1
= 45∫ x 2 (4 − 2 x) 3 dx
x =0 3

= 128
(5) If A = 2 xzi − x j + y 2 k , find ∫∫∫ A dV , where V is the region bounded by
V

x = 0 , y = 0 , y = 6, z =x2 , z = 4
The solution:

2 6 4
∫∫∫ A dV = ∫
V
x =0 ∫ ∫ y =0 z= x2
(2 xzi − x j + y 2 k )dzdy

2 6 4 2 6 4
= i∫ ∫ ∫ 2 xzdzdydx − j ∫ ∫ ∫ xdzdydx
x =0 y =0 z=x 2
0 0 x2
2 6 4
+ k∫ ∫∫ y 2 dzdydx = 128i − 24 j + 38k
0 0 x2

(6) Evaluate ∫∫∫ (2 x + y)dV , where V is the closed region bounded by the surface
V

z + x = 4 and the surfaces x = 0 , y = 0 , y = 2, z=0.


2

The solution:

2 2 4
∫∫∫ (2 x + y)dV = ∫
V
y =0 ∫ ∫
x =0 z =0
(2 x + y )dxdydz

2 2
=∫ ∫ (2 x + y )(4 − x 2 )dxdy
y =0 x =0
2 .
 16 
2  8 
= ∫  8 + y  dy = 8 y + y 2 
y =0
 3   3 0
32 80
= 16 + =
3 3

31
Exercises

First
(1) If A(t ) = 5t 2 i + t j − t 3 k & B(t ) = sin t i − cos t j , find
d
( A(t ) . B(t ) ) & d ( A(t ) ∧ B(t ) ) & d ( A(t ) . A(t ) ) .
dt dt dt

(2) The position vector of a particle is given by r = cos ωt i + sin ωt j , pove that
(i) The velocity v is normal to the position vector r .
(ii)The acceleration a is toward to the origin and proportional to the distance from
the
origin.
(iii) The vector r ∧ v Is costant and find its value.
(3) If A = A(t ) & B = B(t ) & V = V (t ) , prove that
d  dV d 2 V  dV d 3 V
(i) V . ∧ 2  = V . ∧ 3
dt  dt dt  dt dt
d  dB d A  d2B d2 A
(ii)  A∧ − ∧ B = A∧ 2 + B ∧ 2
dt  dt dt  dt dt
(4) If V (x, y, z; t ) = (3 xy + t ) i + (3 x + zt ) k , find
dV
.
dt
Second:
(1) by using elliptic cylindrical coordinates (u , v, z ) ,where
u ≥ 0 & 0 ≤ v < 2π & − ∞ < z < ∞ , x = a cosh u cos v & y = a sinh u sin v & z = z
find the scale factors h1 , h2 , h 3 , find the position Vector and the volume
element.
(2) Find the surface elements of the cylinder ρ = a and the sphere r = a .
(3) by using parabolic coordinates (u , v, φ ) , where u ≥ 0 & v ≥ 0 & − π < φ < π ,

x = a u v cos φ & y = a u v sin φ & z = (u 2 − v 2 ).


a
2
Find the scale factors h1 , h2 , h 3 ,position vector and volume element.
(4) by using the coordinates (ς ,η , φ ) , where ς ≥ 0 & 0 ≤ η ≤ π & 0 < φ < 2π ,
x = a sinη ς sin η cos φ & y = a sinη ς sin η sin φ & z = a cosη ς cosη
Find the unit vectors e ς & eη & e φ .
π π
(5) by using the coordinates (ς ,η , φ ) , where ς ≥ 0 & − & 0 < φ < 2π
≤η ≤
2 2
x = a cosη ς cosη cos φ & y = a cosη ς cosη sin φ & z = a sinη ς sin η .
Find the scale factors ης , ηη , η φ .
Third:

32
π
2
(1) If R(t ) = 3 sin t i + 2 cos t j , find ∫ R(t ) dt
0
2 2
(2) If A(t ) = t i −t j + (t − 1)k & B(t ) = 2t i + 6t k , find ∫ A . B dt & ∫ A ∧ B dt
2 2

0 0
3
dA
(3) If A(2) = 2 i − j + 2k & A(3) = 4i − 2 j + 3 k , find ∫ A . dt
2
dt .

(4) If A = (2 y + 3) i + xz j + ( yz − x) k , find ∫C
A.dr on the following paths,
288
(i) x = 2t 2 & y = t & z = t 3 t : 0 → 1 . ( ).
35
(ii) The straight lines as follows (0,0,0) → (0,0,1) → (0,1,1) → (1,1,1) . (10).
(iii) The straight line from (0,0,0) to (2,1,1) . (8)
(5) Evaluate the closed line integral ∫ F . dr , where F = ( x − 3 y ) i + ( y − 2 x) j , and C is
C

the closed curve in the plane x = 2 cos t & y = 3 sin t & t : 0 → 2π . ( 6π )


(6) Find the work done by the force F = 3x 2 i + (2 xz − y ) j + z k , to move a particle on
the
curve x 2 = 4 y & 3x 3 = 8 z & x : 0 → 2 . (16)
(7) If F = xy i − z j + x k & φ = 2 xyz , and the curve C is x = t 2 & y = 2t & z = t 3 ,
2 2

where t : 0 → 1 , evaluate the following integrals.

8 4   9 2 7 
(i) ∫ φ dr  i + j +k (ii) ∫ F ∧ dr − i − j + k 
C  11 5  C  10 3 5 

(8) If A = 4 xz i + xyz 2 j + 3 z k , evaluate ∫∫ A. dS


S
on the inner surface of the region

above the plane xy and bounded by the cone z 2 = x 2 + y 2 and the plane z = 4 .
( 320π )

∫∫ A. dS , A = z i + x j − 3 y
2
(9) Evaluate z k , where S is the surface of the cylinder
S

x 2 + y 2 = 16 ,in the first octant between the planes z = 0 & z = 5 . (90)


(10) Evaluate ∫∫ F . dS ,
S
if F = 6 z + (2 x + y ) j − x k , and S is the interior surface of the

region bounded by the cylinder x 2 + z 2 = 9 , and the planes x = 0 & y = 0 ,


z = 0 & y = 8. 18π .( )
(11) Evaluate ∫∫ F . dS , where S is the surface of the cylinder which its center is the
S

origin , z is its axis and its height is 2h ,

(i) F = x i + y j ( 4πa 2 h ) (ii) F = x i + y j + z k ( 6πa 2 h )

33
(12) Evaluate ∫∫∫φ dV , where φ = 2 x and V is the bounded region by the planes
V

8
x = 0 & y = 0 & z = 0 , 2x + 2 y + z = 4 ( )
3
∫∫∫ r
2
(13) Evaluate dV , where V is the octant of the sphere
V

1 5
x2 + y2 + z2 ≤ a2 x≥0& y≥0& z≥0 ( πa ) ,
10

(14) Evaluate ∫∫ F . dS
S
,where F = 4 xz i − y 2 j + yz k , and S is the cube surface

3
bounded by the planes x = 0 & x = 1 & y = 0 & y = 1 & z = 0 & z = 1 ( ).
2

(15) Evaluate ∫∫ A. dS ,
S
where A = 18 z i − 12 j + 3 y k and S is that part of the plane

2 x + 3 y + 6 z = 12 , which is located in the first octant. (24)

3
(16) Evaluate ∫∫ φ dS , where φ = 8 xyz
S
and S is the surface of the cylinder x 2 + y 2 = 16

included in the first octant between z = 0 and z = 5 . ( 100 i + 100 j ).

(17) If F = y i + ( x − 2 xz ) j − xy k , evaluate ∫∫ (∇ ∧ F ) . dS , where


S
S is the surface of the

sphere x 2 + y 2 + z 2 = a 2 above the xy − plane. (0).

(18) Evaluate ∫∫ A.dS , for each of the following cases:


S

(i) A = ( x + y 2 ) i − 2 x j + 2 yz k and S is the surface of the plane 2 x + y + 2 z = 16 in


the first octant.
(ii) A = y i + 2 x j − z k and S is the surface of the plane 2 x + y = 6 in the first
octant cut off by the plane z = 4 .

(19) Evaluate ∫∫ (∇ ∧ F ) .dS


S
and ∫∫ φ dS , if
S
F = ( x + 2 y ) i − 3 z j + x k and φ = 4 x + 3 y − 2 z

and S is the surface of the plane 2 x + y + 2 z = 6 bounded by the planes


x = 0 , x = 1 , y = 0 and y = 2 .

(20) If F = (2 x 2 − 3 z ) i − 2 xy j − 4 x k , evaluate ∫∫∫ (∇ . F ) dV


V
and ∫∫∫ (∇ . ∧ F ) dV , where
V

V is the closed region bounded by the planes x = 0 . y = 0 , z = 0 and 2 x + 2 y + z = 4 .

34
CHAPTER II

(Vector Operators)

FIRST: -Gradient of Scalar Function in O.C.C

If φ is a scalar function and P( u 1 , u 2 , u 3 ) is a point in space , and suppose that


e 1 , e 2 , e 3 are the unit vectors in the directions of the tangents for curves u 1 , u 2 , u 3
respectively . The position vector r is a function of (u 1 , u 2 , u 3 ) , i.e.
r = r (u1 , u 2 , u3 ) ⇒ d r = h1du1 e1 + h2 du 2 e 2 + h3 du3 e 3 (1)
where h1 , h2 , h3 are the scale factors.
∴φ = φ (u1 , u 2 , u3 )
∂φ ∂φ ∂φ
dφ = du1 + du 2 + du3
∂u1 ∂u 2 ∂u3
h1 ∂φ h ∂φ h ∂φ
= du1 + 2 du 2 + 3 du3
h1 ∂u1 h2 ∂u 2 h3 ∂u3
 1 ∂φ 1 ∂φ 1 ∂φ 
=  e1 + e2 + e 3 .(h1du1 e1 + h2 du 2 e 2 + h3 du3 e 3 ) (2)
 h1 ∂u1 h2 ∂u 2 h3 ∂u3 
From (1) in (2)
 1 ∂φ 1 ∂φ 1 ∂φ 
dφ =  e1 + e2 + e 3 .d r (3)
 h1 ∂u1 h2 ∂u 2 h3 ∂u3 
let
1 ∂φ 1 ∂φ 1 ∂φ
∇φ = e1 + e2 + e3
h1 ∂u1 h2 ∂u 2 h3 ∂u3
and dφ = ∇φ .d r

Remarkes:-

(1) The operator ∇ (Grad or Nabla or Del) in the orthogonal curvilinear coordinates
e ∂ e ∂ e ∂
∇= 1 + 2 + 3 (I)
h1 ∂u1 h2 ∂u 2 h3 ∂u 3
In the Cartisan coordinates
∂φ ∂φ ∂φ
∇φ = i+ j+ k : φ is any scalar function
∂x ∂y ∂z
In the cylindrical coordinates
∂ψ 1 ∂ψ ∂ψ
∇ψ = eρ + eφ + ez
∂ρ ρ ∂φ ∂z
In the spherical coordinates

35
∂ψ 1 ∂ψ 1 ∂ψ
∇ψ = er + eθ + eφ
∂r r ∂θ r sin θ ∂φ
(2) The gradient of the function φ(u 1 , u 2 , u 3 ) in the direction of the unit vector a is
Of the form ∇φ.a and is called the directional derivative of φ in the direction a .
(3) ∇φ is normal to the surface φ = C .

Proof:
We know that φ = C ⇒ dφ = 0 ⇒ ∇φ . dr = 0
Then ∇φ ⊥ d r but dr lies in the tangent plane to the surface at P.
Then ∇φ is perpendicular to the surface.
And the normal unit vector n on the surface φ = c is of the fom
∇φ
n=
∇φ
(4) The operator A.∇ is the scalar operator and is defined as
A ∂ A ∂ A ∂
A.∇ = 1 + 2 + 3 (II)
h 1 ∂u 1 h 2 ∂u 2 h 3 ∂u 3
We notice that
A ∂φ A 2 ∂φ A 3 ∂φ
a ) (A.∇)φ = A.∇φ = ∇φ.A = 1 + +
h 1 ∂u 1 h 2 ∂u 2 h 3 ∂u 3
A1 ∂ B A2 ∂ B A3 ∂ B
b) ( A.∇) B = + +
h1 ∂u1 h2 ∂u 2 h3 ∂u 3
 A ∂B A2 ∂B A3 ∂B   A ∂B A2 ∂B A3 ∂B 
=  1 + +  e1 +  1 + +  e 2
 h1 ∂u1 h2 ∂u 2 h3 ∂u 3   h1 ∂u1 h2 ∂u 2 h3 ∂u 3 
 A ∂B A2 ∂B A3 ∂B 
+  1 + +  e 3
 1 1
h ∂u h2 ∂u 2 h3 ∂u 3 

c) A . ∇ ≠ ∇ . A

The operator ∇ is operate on a scalar not on a vector, but the operator (A.∇) operate
on a scalar or a vector.

(5) If x, y, z functions of t (time) i.e φ = φ( x , y, z, t )


dφ ∂φ dx ∂φ dy ∂φ dz ∂φ
= + + +
dt ∂x dt ∂y dt ∂z dt ∂t
 ∂φ ∂φ ∂φ   dx dy kz  ∂φ
=  i + j+ k . i + j + k+
Then  ∂x ∂y ∂z   dt dt dt  ∂t
∂φ dx dy kz
= (∇φ .V ) + V = i+ j + k = Vx i + V y j + Vz k
∂t dt dt dt
∂φ
= (V .∇)φ +
∂t

36
d ∂
In general = (V .∇) +
dt ∂t
(6) For any two scalar functions φ , ψ
∇(φψ ) = φ∇ψ + ψ∇φ
Proof:
e ∂ e ∂ e ∂
∇(φψ ) = 1 (φψ ) + 2 (φψ ) + 3 (φψ )
h1 ∂u1 h2 ∂u 2 h3 ∂u3
e1  ∂ψ ∂φ  e 2  ∂ψ  ∂ψ ∂φ 
∂φ  e 3
= φ +ψ + φ φ+ψ +ψ + 
h1  ∂u1 ∂u1  h2  ∂u3
 ∂u 2 ∂u3 
∂u 2  h3
 e ∂ψ e 2 ∂ψ e 3 ∂ψ   e1 ∂φ e 2 ∂ψ e 3 ∂ψ 
=φ 1 + +  +ψ  + + 
 h1 ∂u1 h2 ∂u 2 h3 ∂u3   h1 ∂u1 h2 ∂u 2 h3 ∂u3 
= φ∇ψ +ψ∇φ

Solved examples:

(1) If φ ( x, y, z ) = 3x 2 y − y 3 z , find grad φ at the point (1, −2, −1)


Solution:
∂φ ∂φ ∂φ
grad φ = ∇φ = i+ j+ k
∂x ∂y ∂z
∴ ∇φ = 6 xyi + (3 x 2 − 3 y 2 z ) j − y 3 k
∴ ∇φ (1, −2, −1) = −12i + 16 j + 8k

(2) If ψ (r ,θ , φ ) = r 2 cos θ sin nφ , find grad ψ

Solution:
∂ψ 1 ∂ψ 1 ∂ψ
gradψ = er + eθ + eφ
∂r r ∂φ r sin θ ∂φ
= 2r cosθ sin nφ e r − r sin θ sin nφ eθ + nr cot θ cos nφ eφ

(3) Find the directional derivative for the function φ = x 2 y + y 2 z + xz 2 in the direction
of the vector A = 3i + j − 3k .

Solution:
^ A (3,1,−3) 1
A= = = (3,1,−3)
| A| 9 +1+ 9 19
^
A is the unit vector in the direction of A

∇φ = (2 xy + z 2 )i + (2 yz + x 2 ) j + (2 xz + y 2 )k
1
[3(2 xy + z ]
^
⇒ ∇φ , A = 2
) + (2 yz + x 2 ) − 3(2 xz + y 2 )
19

37
a
(2) If A = e r + sin θ eθ , B= sin θ e r , find ( A.∇) B
r2
Solution:
 a 
( A.∇) B = [(e r + sin θ eθ ).∇] 2 sin θ e r 
r 
 ∂ sin θ ∂  a 
= +  2 sin θ e r 
 ∂r r ∂θ  r 
2a a ∂e r a a sin 2 θ ∂ e r
=− sin θ e + sin θ + sin θ cos θ e +
∂r r 3 ∂θ
r r
r3 r2 r3
∂e r ∂e r
Since =0, = eθ
∂r ∂θ
a sin θ a sin θ
Then ( A.∇) B = 3
(cosθ − 2)e r + eθ
r r3

(5) Prove that ∇r n = nr n −2 r


Solution:
Let ψ = r n , and by using spherical coordinate
∂ r r
∇ψ = ∇r n = r n = nr n −1 e r = nr n −1 = nr n − 2 r : er =
∂r r r

(6) Find the normal unit vector on the surface x 2 y + 2 xz = 4 at the point (2, −2, 3).

Solution:
Let φ( x , y, z) = 4 , where φ = x 2 y + 2 xz − 4 ,
∇φ
Then the normal unit vector is n =
∇φ
∇φ = (2 xy + 2z)i + x 2 j + 2 x k

∇φ ( 2, −2,3) = −2i + x 2 j + 4k | ∇φ |= 36 = 6
1 2 2
⇒ n = − i + j+ k
3 3 3

We notice that (− n ) is also normal unit vector but in opposite direction.

(7) Find the angle between the surfaces z = x 2 + y 2 − 3 , x 2 + y 2 + z 2 = 9 at the point


( 2,−1,2)

Solution:
The angle between the surfaces at any points is the angle between the normal on the
surfaces at this point.

38
∇φ1 .∇φ2 ∇φ1 ∇φ2
cosθ = = = n1 .n 2
|| ∇φ1 |||| ∇φ2 || || ∇φ1 || || ∇φ2 ||
where φ1 = x 2 + y 2 + z 2 , φ2 = x 2 + y2 − z
∇φ 2 , ∇φ1 are the normal at the surfaces φ 2 , φ1 respectively.

and n 2 , n 1 are the normal unit vectors on the surfaces φ 2 , φ1 .

∇φ1 = 2 xi + 2 y j + 2 z k ∇φ1 ( 2, −1, 2 ) = 4i − 2 j + 4k


∇φ 2 = 2 x i + 2 y j − k ∇φ 2 ( 2 , −1, 2 )
= 4i − 2 j − k
∇φ 1 2 1 2 1
n1 = = i − j + k = (2,−1,2)
|| ∇φ1 || 3 3 3 3

∇φ 2 1 1
n2 = = (4i − 2 j − k ) = (4,−2,−1)
|| ∇φ 2 || 21 21
1 8
then cos θ = n1 .n 2 = (8 + 2 − 2) = ~ 0.582
3 21 3 21
∴ θ ~ cos −1 (0.582) ~ 54 o 25'

Exercises

(1) If φ & ψ are the differentiable scalar functions in the cartisean coordinates x, y, z
prove that
(i) ∇(φ ± ψ ) = ∇φ ± ∇ψ
 φ  ψ∇φ − φ∇ψ
(ii) ∇  = .
ψ  ψ2
1
(2) Find ∇φ , if φ = & φ = ln r
r
(3) Find the tangential plane for the surface 2 xz 2 − 3 xy − 4 x = 7 , at the point ( x0 , y 0 , z 0 ) .
(4) Find the directional derivative for the function φ ( x, y, z ) = x 2 yz + 4 xz 2 in the
direction of the vector 2 i − j − 2k .
(5) If a is a constant vector , prove that ∇(a . r ) = a .
f (r )
(6) Prove that ∇f (r ) = r.
r
(7) If f = f ( x, y, z ) , prove that df = ∇f . dr .
(8) Find the acute angle between the surfaces xy 2 z = 3 x + z 2 & 3 x 2 − y 2 + 2 z = 1 at the
point (1,−2,1) .
(9) Find ∇ψ for the following:
1
(i) ψ ( ρ , φ ) = ρ cos ϕ − sin ρ
ρ

39
1
(ii) ψ ( ρ , θ , ϕ ) = (r 2 + ) cos θ sin φ .
r
(10) by using elliptic cylindrical coordinates (u, v, z ) , where
u ≥ 0 & 0 ≤ v < 2π & − ∞ < z < ∞ , x = a cosh u cos v & y = a sinh u sin v & z = z
find gradψ ,where ψ = ψ (u, v, z ) .
(11) Prove that:
(i) In cylindrical coordinates (ρ , φ , z ) ∇ρ = e ρ & ∇φ = e φ & ∇z = e z
1
ρ
(ii) In spherical coordinates (r , θ , φ ) ∇r = e r & ∇θ = eθ & ∇φ =
1 1
e φz .
r r sin θ
(12) If f = f ( x, y, z ) is a scalar function where, f ( x, y, z ) = x 2 y + e z , find ∇f at any point
,and then find the directions cosines at the point (1,5,0) .
(13) Find the constants a, b ,if the surface ax 2 − byz = (a + 2) x is normal to the surface
4 x 2 y + z 3 = 4 , at the point (1,−1,2)

SECOND: (Vector Field Divergence).


∫∫ A.d S
The field divergence of a vector A at is div A = lim , S
∆V →0 ∆V
where ∆V is the volume element which contain the point P, and its surface area is ∆S.
To find (div A ) in the orthogonal curvilinear coordinates. Let the volume element dV
is of the form(see fig.) has the sides h 1 du 1 , h 2 du 2 , h 3 du 3
Let the vector A is of the form
A = A1 e1 + A2 e 2 + A3 e 3
and n is the outward normal unit vector on the surface dS which contains the volume
dV .
we notice that the normal on the surface ABCD is n = e 1 .

∫∫ A.d s = ∫∫ A.n d s = ∫∫ [e . A]ds


ABCD ABCD ABCD
1

= A1h2 h3 du 2 du3

Similarly

∫∫ A.d s = − A h h du du
EFGH
1 2 3 2 3 +
∂u1
( A1h2 h3 du 2 du3 )du1 + O(u1 , u 2 , u3 )


The value of integral on the surfaces EFGH , ABCD is ( A1h2 h3 du 2 du3 )du3
∂u1
Similarly ,we can find the integrals on the surfaces, and the integral on the all surface is

40
 ∂ ∂ ∂ 
∫∫ A.d S =  ∂u
S 1
( A1 h2 h3 ) +
∂u 2
( A2 h1 h3 ) +
∂u 3
( A3 h1 h2 ) du1 du 2 du 3

Dividing by the volume element dV = h1h2 h3 du1du 2 du3 .
1  ∂ ∂ ∂ 
div A =  ( A1 h2 h3 ) + ( A2 h3 h1 ) + ( A3 h1 h2 ) = ∇. A` (III)
h1 h2 h3  ∂u1 ∂u 2 ∂u 3 

We can prove (III) by another way.

Let A = A1 e1 + A2 e 2 + A3 e 3
So
∇. A = ∇.( A1 e1 + A3 e 3 + A3 e 3 )
e ∂ e ∂ e ∂  (1)
=  1 + 2 + 3 .( A1 e1 + A3 e 3 + A3 e 3 )

 1 1
h u h2 ∂u 2 h3 ∂u 3 

but
∂ e1 1 ∂h1 1 ∂h1 ∂ e1 1 ∂h2 ∂ e1 1 ∂h3
=− e2 − e2 , = e2 , = e3 (2)
∂u1 h2 ∂u 2 h3 ∂u3 ∂u 2 h1 ∂u1 ∂u3 h1 ∂u1
From (2) in (1)
1 ∂A1 A ∂h2 A ∂h3 1 ∂
∇.( A1.e1 ) = + 1 + 1 = (h2 h3 A1 )
h1 ∂u1 h1h2 ∂u1 h1h3 ∂u1 h1h2 h3 ∂u1
Similarly
1 ∂ 1 ∂
∇.( A2 e 2 ) = (h1h3 A2 ), ∇.( A3 e 3 ) = (h1h2 A3 )
h1h2 h3 ∂u 2 h1h2 h3 ∂u3
1  ∂ ∂ ∂ 
Then ∇. A =  ( A1h2 h3 ) + ( A2 h1h3 ) + ( A3 h1h2 )
h1h2 h3  ∂u1 ∂u 2 ∂u3 

We notice that from ()II( ,)III) that A.∇ , ∇.A .

Remarks:
(1) If (∇.A = 0) then the vector A is called Solenoidal.
(2) The operator ∇ 2 or ∇.∇ is called (Laplacian), then for any scalar function φ
div gradφ = ∇.(∇φ ) = ∇ 2φ
1  ∂  h2 h3 ∂φ  ∂  h3 h1 ∂φ  ∂  h1h2 ∂φ 
=    +   +  
h1h2 h3 ∂
 1 1
u h ∂u1  ∂u 2  2
h ∂u 2  ∂u3  3
h ∂u 3 

And the differential equation ∇ 2 φ = 0 , is called Laplace equation and the function

φ is called Harmonic function and is of the form

∂ 2φ ∂ 2φ ∂ 2φ
∇ φ=
2
+ +
∂x 2 ∂y 2 ∂z 2

41
(3) For any scalar function φ , ∇.(φA) = φ∇.A + A.∇φ
Proof:
1  ∂ ∂ ∂ 
∇.(φ A) =  (φA1h2 h3 ) + (φA2 h1h3 ) + (φA3 h1h2 )
h1h2 h3  ∂u1 ∂u 2 ∂u3 
φ  ∂ ∂ ∂ 
=  ( A1h2 h3 ) + ( A2 h1h3 ) + ( A3 h1h2 )
h1h2 h3  ∂u1 ∂u 2 ∂u3 
 A ∂φ A2 ∂φ A3 ∂φ 
+  1 + +  = φ∇. A + A.∇φ

 1 1
h u h 2 ∂u 2 h3 ∂u 3 

Examples:
 r 
(1) Prove that ∇. =0
 r3 
Solution:
 r 
∇. 3  = ∇.(r −3 r ) = r −3∇.r + r.∇r −3 = 3r −3 + r.(−3r −5 r )
r 
= 3r −3 − 3r −5 r 2 = 3r −3 − 3r −3 = 0

Another solution by using spherical coordinates:


r 1 1
Let A = 3 = 2 e r = Ar e r : Ar = 2
r r r
1 ∂ 2
⇒ div A = 2 (r sin θAr ) + 0 + 0
r sin θ ∂r
1 ∂ 1 ∂   1   1 ∂1
= 2 sin θ ( Arr 2 ) = 2  r 2  2   = 2 =0
r sin θ ∂r r ∂r   r   r ∂r
1
(2) Prove that ∇ 2   = 0 .
r
Solution:
1 −r  r 
∇ 2   = ∇ 2 (r −1 ) = ∇.∇r −1 = ∇. 3 = − ∇. 3  = 0
r r  r 

(3) Prove that ∇ 2 r n = n(n + 1)r n−2 .

Solution: by using the spherical coordinates:


Let ψ = rn
1 ∂ 2 n ∂ n +1 n
∴∇2r n = 2 r (nr n −1 ) = 2 (r ) = 2 (n + 1)r n = n(n + 1)r n − 2
r ∂r r ∂r r

(4) Prove that ∇.(u∇v − v∇u ) = u∇ 2 v − v∇ 2 u

42
Solution:
∇.(u∇v) = u∇ 2 v + ∇u.∇v
∇.(v∇u ) = v∇ 2 u + ∇v.∇u = v∇ 2 u + ∇u.∇v
∇.(u∇v − v∇u ) = u∇ 2 v + ∇u.∇v − v∇ 2 u − ∇u.∇v = u∇ 2 v − v∇ 2 u

(5) Find the constants λ if the vector V is solenoid.

V ( x, y, z ) = ( x + 3 y )i + ( y − 2 x) j + ( x + λz )k
Solution:
If the vector is solenoidal then ∇.V = 0

∂Vx ∂Vy ∂Vz


∇.V = + + = 1+1+ λ = 0
∂x ∂y ∂z
⇒ λ = −2

 r 
(6) Evaluate ∇ 2 ∇. 2 
 r 
Solution:
r
 ∇. 2 = ∇.r −2 r = r −2 (∇.r ) + ∇r −2 r.r
r
= 3r −2 − 2r −4 r 2 = r −2
 r  2
∴ ∇ 2 ∇. 2  = ∇ 2 (r −2 ) = −2(2 + 1)r −4 = 4
 r  r

(7) Prove that ∇ 2 (φψ ) = φ∇ 2ψ + 2∇φ .∇ψ + ψ∇ 2φ


Solution:

∇ 2 (φψ ) = ∇.∇(φψ )
= ∇.(φ∇ψ + ψ∇φ )
= φ∇ 2ψ + ∇φ .∇ψ + ∇ψ .∇φ + ψ∇ 2φ
= φ∇ 2ψ + 2∇φ .∇ψ + ψ∇ 2φ
(8) If b , a are constant vectors.
div[(r ∧ a ) ∧ b] = −2(b.a )
Solution:

div[(r ∧ a ) ∧ b] = div[(r.b)a − (a.b)r ]


= ∇.[(r.b)a − (a.b)r ] (1)
= (r.b)∇.a + ∇(r.b).a − (a.b)∇.r + ∇(a.b).r

43
∇(r.b) = ∇(b1 x + b2 y + b3 z ) : b = (b1 , b2 , b3 ) , r = ( x, y, z )
(2)
= b1 i + b2 j + b k = b
From (2) in (1) we get
: div[(r ∧ a ).b] = b.a − 2(a.b) = b.a − 3(b.a ) = −2b.a

Exercises

(1) For any Vector fields A & B and any scalar function φ , prove that
(i) ∇ . ( A + B ) = ∇ . A + ∇ . B
(ii) ∇ . (φ A) = φ (∇ . A) + ∇φ . A
(iii) ∇ 2 (φψ ) = φ∇ 2ψ + 2∇φ . ∇ψ + ψ∇ 2φ .

(2) Prove that:


(i) ∇ . r = 3
(ii) ∇ . (r 3 r ) = 6r 3
  r  2
(iii) ∇ 2 ∇ .  2  = 4
  r  r
  1  3
(iv) ∇ . r ∇  3  = 4
  r  r
(v) ∇ 2 (ln r ) = 2 .
1
r
(3) Prove that the vector field A = (2 x 2 + 8 xy 2 z ) i + (3 x 3 y − 3 xy ) j − (4 y 2 z 2 + 2 x 3 z ) k is a
solenoid vector, also the vector field B = xyz 2 A .
(4) (i) If ψ = ψ ( ρ , φ , z ) in cylindrical coordinates, prove that
1 ∂  ∂  1 ∂2 ∂2
∇ 2ψ =  ρ (ψ )  + + (ψ )
ρ ∂ρ  ∂ρ  ρ ∂φ
2
∂z 2
(ii) If ψ = ψ (r , θ , φ ) in spherical coordinates, prove that
1 ∂  ∂  1 ∂  ∂  1 ∂2
∇ 2ψ = 2  r (ψ )  + 2  sin θ (ψ )  + (ψ )
r ∂r  ∂r  r sin θ ∂θ  ∂θ  r sin θ ∂φ
2 2 2

(5) Find ∇ψ and ∇ . A in elliptic cylindrical coordinates (u, v, z ) , where


u ≥ 0 & 0 ≤ v < 2π & − ∞ < z < ∞ x = a cosh u cos v & y = a sinh u sin v & z = z .
(6) Find ∇ ψ in the following cases:
2

(i) ψ (r , θ , φ ) = r cos nθ cos mφ


−z
(ii) ψ ( ρ , θ , φ ) = e ln ρ cos φ .
(7) From Gauss divergent theorem, prove that:
 ∂ψ ∂φ 
(i) ∫∫  φ −ψ  dS = ∫∫∫ (φ∇ ψ − ψ∇ φ )dV
2 2

S 
∂n ∂n  V

(ii) ∫∫ r ∧ dS = 0

44
(iii) ∫∫∫ (∇ ∧ B) dV = 0 , where B is always perpendicular to the closed surface S .
(iv) ∫∫ n dS = 0 , where n is the unit outward vector perpendicular to the closed
S

surface S .
1 r .n
(v) ∫∫∫ r
V
2
dV = ∫∫
S r2
dS

(vi) ∫∫ r . dS = 32π , where S is the sphere of radius 2 and centered at the origin.
S

(vii) ∫∫ r . dS = 24
S
, where S is the cube confined by the planes

x = ±1 & y = ±1 and z = ±1 .
(viii) ∫∫ ∇ ∧ A . dS = 0 , for any vector field
S
A.

45
Third: (The Curl of a Vector Field)
The curl of the vector A at any point P is a vector and is in the normal direction
on S.

∫ A.d r
(curl A).n = lim C
∆S →0 ∆S

where ∆S is the area which contain the point P and is bounded by C, n is the normal
unit
vectors on ∆S. and is denoted by rot A , curl A , or ∇ ∧ A .
The definition of curl A in orthogonal curvilinear coordinates is in the form

h1 e1 h2 e 2 h3 e 3
1 ∂ ∂ ∂
∴ curl A = =∇∧ A (*)
h1h2 h3 ∂u1 ∂u 2 ∂u3
h1 A1 h2 A2 h3 A3

We can deduce (*) by using another way as follows:

e ∂ e ∂ e ∂ 
 ∧ ( A1 e1 + A2 e 2 + A3 e 3 )
1
∇∧ A= =  1 + 2 + 3
h1h2 h3  h1 ∂u1 h2 ∂u 2 h3 ∂u3 
We calculate the term ∇ ∧ (A 1 e 1 ) only as follows:
e1 ∂ e ∂ e ∂
∇ ∧ ( A1 e1 ) = ∧ ( A1 e1 ) + 2 ∧ ( A1 e1 ) + 3 ∧ ( A1 e1 )
h1 ∂u1 h2 ∂u 2 h3 ∂u 3
e1  ∂A1 ∂e  e  ∂A ∂e  e 3  ∂A1 ∂e 
= ∧  e1 + A1 1  + 2 ∧  e1 1 + A1 1  + ∧  e1 + A1 1 
h1  ∂u1 ∂u1  h2  ∂u 2 ∂u 2  h3  ∂u 3 ∂u 3 
A1  1 ∂h1 1 ∂h1  1 ∂A1 1 ∂A1
=  − e3 − e 2  − e3 + e2
h1  h2 ∂u 2 h3 ∂u 3  h2 ∂u 2 h3 ∂u 2
 A ∂h1 1 ∂A1   A ∂h1 1 ∂A1 
=  1 +  e 2 −  1 +  e 3
 2 3 3
h h ∂ u h3 ∂u 3   1 2 2
h h ∂ u h 2 ∂ u 2 

e ∂ e ∂
= 1 ( A1 h1 ) − 1 ( A1 h1 )
h1 h3 ∂u 3 h1 h2 ∂u 2
Similarly
1  e3 ∂ e ∂ 
∇ ∧ ( A2 e 2 ) =  ( A2 h2 ) − 1 ( A2 h2 )
h2  h1 ∂u1 h3 ∂u 3 
1  e1 ∂ e ∂ 
∇ ∧ ( A3 e 3 ) =  ( A3 h3 ) − 3 ( A3 h3 )
h3  h3 ∂u 2 h1 ∂u1 
By adding

46
e1  ∂ ∂ 
∇∧ A=  ( A3 h3 ) − ( A2 h2 )
h2 h3 ∂u 2 ∂u 3 
e  ∂ ∂  e  ∂ ∂ 
+ 2  ( A1h1 ) − ( A3 h3 ) + 3  ( A2 h2 ) − ( A1 h1 )
h1h3  ∂u3 ∂u1  h1 h2  ∂u1 ∂u 2 
h1 e1 h2 e 2 h3 e3
1 ∂ ∂ ∂
=
h1h2 h3 ∂u1 ∂u2 ∂u3
h1 A1 h2 A2 h2 A2

Remarks:
(1) If ∇ ∧ A = 0 ⇔ A is called irrotational.

(2) for any scalar function φ , curl grad φ = ∇ ∧ (∇φ ) = 0 .


(3) for any vector field , div curl A = ∇.(∇ ∧ A) = 0 .
(4) for any scalar function φ , ∇ ∧ (φ A) = φ∇ ∧ A + ∇φ ∧ A

Solved examples:
(1) If A = xz 3 i − 2 x 2 y j + 2 yz 4 k , find curl A at the point (1,−1,1).
Solution:
 A = A( x, y, z )
i j k
∂ ∂ ∂ ∂ ∂  ∂ ∂ 
∴∇ ∧ A = =  (2 yz 4 ) − (−2 x 2 yz )i +  ( xz 3 ) − (2 yz 4 ) j
∂x ∂y ∂z  ∂y ∂z   ∂z ∂x 
xz 3 − 2 x yz 2 yz
2 4

= (2 z 4 + 2 x 2 y )i + 3 xz 2 j − 4 xyz k
Then ∇ ∧ A (1, −1,1) = 3 j + 4k
(2) find the constants a, b, c , if the vector A is irrotational .
A = ( x + 2 y + az )i + (bx − 3 y − z ) j + (4 x + cy + 2 z )k
Solution:where

∇∧ A=0
i j k
∂ ∂ ∂

∂x ∂y ∂z
x + 2 y + az bx − 3 y − z 4 x + cy + 2 z
= (c + 1)i + (a − 4) j + (b − 2)k ⇒ c = −1 , a = 4 , b=2

(3) prove that for any scalar function φ, curl (φ grad dφ) = 0
Solution:

47
curl (φ grad φ) = ∇ ∧ (φ∇φ) = ∇ ∧ (φA) : A = ∇φ
Then
curl(φ grad φ) = φ∇ ∧ A + ∇φ ∧ A

= φ∇ ∧ ∇φ + ∇φ ∧ ∇φ
= 0+0 = 0
(4) prove that if the scalar function φ is a solution of Laplace’s equation then
∇φ is solinoidal and irrotational.

Solution:
If φ is the solution of Laplace’s equation then
∇.∇φ = 0 ⇐ ∇ 2φ = 0

then φ is solinoidal and ∇φ is irrotational.

 r 
(5) prove that ∇∧ 2  = 0.
r 
Solution:
 1 
∇ ∧  2 r  = ∇ ∧ (r −2 r )
r 
= r − 2 (∇ ∧ r ) + ∇r − 2 ∧ r [using (∇ ∧ φ A)]
since ∇ ∧ r = 0
and ∇r 2 = nr n −2 r
 r 
∴ ∇ ∧  2  = 0 − 2r − 4 r ∧ r = 0 since r ∧ r = 0
r 
 r 
∴∇ ∧  2  = 0
r 

(6) If b , a are constant vectors, prove that


curl [(r ∧ a ) ∧ b] = b ∧ a .
Solution:
curl [(r ∧ a ) ∧ b] = curl [(r.b)a − (a.b)r ]
= [(r.b)a − (a.b)r ]
= (r.b)∇ ∧ r − ∇(a.b) ∧ r
=b∧a
where ∇ ∧ a = ∇ ∧ r = 0 , ∇(r.b) = b

Gauss Divergence Theorem:


If S is a closed curve which contain a volume V and the unit vector n is normal on
the
surface S and is in the outward direction from the volume V .

48
‫" و�ﺎن اﻟمتجﻪ ‪ A‬ﻤﻌرﻓًﺎ ﻋند �ﻞ ﻨﻘطﺔ ﻓﻲ اﻟحجم ‪ V‬واﻟسطﺢ ‪ ,S‬ﻓﺈن ف�ض اﻟمتجﻪ ‪ A‬ﻋبر اﻟسطﺢ اﻟمﻐﻠق ‪S‬‬
‫�سﺎوى ﺘكﺎﻤﻞ اﻨتشﺎر ﻫذا اﻟمتجﻪ ﻋﻠﻰ اﻟحجم ‪ V‬اﻟواﻗﻊ داﺨﻞ ذﻟك اﻟسطﺢ ‪" S‬‬
‫أي أن‬

‫‪∫∫∫ div A dV = ∫∫ A.dS‬‬


‫‪V‬‬ ‫‪S‬‬

‫اﻟبرﻫﺎن‪:‬‬
‫ﻨﻘسم اﻟحجم إﻟﻰ ﻋنﺎﺼر ﺼﻐیرة �ﻞ ﻤنﻬﺎ ‪ .∆V‬وﻤن ﺘﻌر�ف اﻨتشﺎر اﻟمتجﻪ ﻏیر اﻟسـطﺢ ‪) ∆S‬اﻟمﻐﻠـق ﻟﻠحجـم ‪(∆V‬‬
‫ﻨجد أن‬
‫= ‪div A ∆V‬‬
‫‪∫∫ A .dS‬‬
‫‪∆S‬‬
‫)‪(1‬‬

‫�ﺎﻟمثﻞ �ﺎﻟنس�ﺔ ﻟ�ق�ﺔ اﻟﻌنﺎﺼر اﻷﺨرى ﻤن اﻟسطﺢ ف�كون‬

‫‪∑ div A ∆V = ∑ ∫∫ A.dS‬‬ ‫)‪(2‬‬


‫‪V‬‬ ‫‪∆S‬‬

‫ﺒینمـﺎ ﻨﻼﺤـظ ﻓـﻲ اﻟطـرف اﻷ�مـن أن اﻟسـطوح اﻟداﺨﻠ�ـﺔ ﻟﻸﺤجـﺎم‬


‫اﻟطرف اﻷ�سر ﻟﻠمﻌﺎدﻟﺔ )‪ (2‬ﻫـو ‪∫∫∫ div A.dV‬‬
‫‪V‬‬

‫ﺘتكرر �ﺈﺸﺎرات ﻤضﺎدة و�ـذﻟك ﺘتﻼﺸـﻰ �ﻠﻬـﺎ أﺜنـﺎء ﻋمﻠ�ـﺔ اﻟجمـﻊ و�ت�ﻘـﻰ اﻟسـطﺢ اﻟخـﺎرﺠﻲ ‪ S‬ﻓﻘـط وﻋﻠـﻰ ذﻟـك ﯿـؤول‬
‫‪.‬‬
‫اﻟطرف اﻷ�من إﻟﻰ ﺘكﺎﻤﻞ ﻋﻠﻰ اﻟسطﺢ ‪� S‬ﺎﻟتﺎﻟﻲ ‪∫∫ A.dS‬‬
‫‪S‬‬

‫وﻫكذا ﺘص�ﺢ اﻟمﻌﺎدﻟﺔ )‪ (2‬ﻓﻲ اﻟصورة‬

‫‪∫∫∫ div A dV = ∫∫ A dS‬‬


‫‪V‬‬ ‫‪S‬‬

‫و�ذﻟك ﻨكون أﺜبتنﺎ اﻟنظر�ﺔ )وﻫو اﻟمطﻠوب(‪.‬‬

‫‪Results:‬‬
‫‪(1) If A = ∇ ∧ a‬‬ ‫⇒‬
‫‪∫∫ A.dS = ∫∫∫ ∇.(∇ ∧ a)dV = 0‬‬
‫‪S‬‬ ‫‪V‬‬

‫‪then‬‬
‫‪∫∫ A.dS = 0‬‬
‫‪S‬‬

‫⇒ ‪(2) If A = r : r is the position vector‬‬


‫‪∫∫ r.dS = ∫∫∫ ∇.rdA = 3V‬‬
‫‪S‬‬ ‫‪V‬‬

‫‪∫∫ r.dS‬‬
‫‪1‬‬
‫= ‪∴V‬‬
‫‪3‬‬
‫‪S‬‬
‫‪(3) If A = n‬‬
‫‪where n is the normal unit vector on the surface S.‬‬

‫‪49‬‬

∫∫∫ ∇.n dV = S
V

Solved problems:
(1) by using Gauss divergence theorem, evaluate ∫∫ F.dS , where
S
F = x i − xy j + 3z k and the surface S is the interior surface of a half sphere
which
. radius is 2 in the xy plane.

Solution:
We have
∫∫ F.dS = ∫∫∫ div F dV
S V
Where
div F = 1 − x + 3 = 4 − x
= 4 − r sin θ cos φ
dV = r 2 sin θdrdθdφ
π
2 2π
∴ ∫∫∫ div F dV = ∫ 2
∫ ∫ (4 − r sin θ cos φ )r 2 sin θdrdθdφ
r =0 θ =0 φ =0
V
π π
2 2π 2 2π
= 4∫ ∫ ∫
2 r 2 dr sin θdθdφ − ∫ ∫ ∫
2 r 3 sin 2 θ cos φdrdθdφ
0 0 0 0 0 0
2 π
r 2  π
8 64π
= 4   [− cos θ ] 2 [φ ] 0 − ∫ ∫ 2 r 3 sin 2 θ ∫ cos φdφ = 4 (1)(2π ) =
2π 2 2π

 3 0 0 0 0
3 3

1
3 ∫∫
(2) prove that for any closed surface S which contain a volume V then V = r.d S .
S

and using this result to obtain the volume of right cone which its radius a and its
height h.

Solution:
From Gauss theorem,

∫∫ A.d S = ∫∫∫ div A dV


S V

Let A = r we get

1
3 ∫∫
V= r.d S (1)
S

50
For a right cone the surface S consist of two parts

(a) The cone base

where 0 < ρ < a , 0 < φ < 2π, z = h , ds 1 = ρdρdφ


ds = nds 1 , n 1 = e z
a 2π
∴ ∫∫ r.d s = ∫ ∫ ( ρ cos φ e ρ + ρ sin φ e φ + z e z ).e z ( ρdρdφ )
ρ =0 φ =0
S1
a 2π
= h∫ ∫ ρdρdφ = πa 2 h (2)
0 0

(b) The cone surface (S 2 ) by using spherical coordinates.


r = rer , ds 2 = r 2 sin αdrdφ , n 2 = eθ
r.d s = r.n 2 ds 2 = 0 ∴ ∫∫ r.d s = 0 (3)
S2

By adding (2) and (3)

∫∫ r.d s = πa
2
h (4)
S

1 2
Substitute from (4) in (1) we get V = πa h
3
(3) Evaluate the surface integral
∫∫ F.ds , where F = x( zi + y j + z k ) and S is the
S
surface
Which contain the volume of the upper half of a sphere
x2 + y2 + z2 ≤ a 2 , z ≤ 0 . Verify the result by using Gauss theorem.
Solution:

First: by using spherical coordinates

x = r sin θ cos φ
y = r sin θ sin φ
z = r cosθ

On the surface S1

51
r
ds1 = n ds, n = er =
r
ds = r 2 sin θdθdφ
1
∴ d s 1 = ( xi + y j + z k )ds
r
1
∴ F .d s1 = ( x 2 z + xy 2 + xz 2 )r 2 sin θdθdφ
r

On the surface S1
π
r = a, 0 < θ < , 0 < φ < 2π
2
π
∴ ∫∫ F .d s1 = a 4 ∫ 2 ∫
0

0
[sin 3
θ cos q cos 2 φ + sin 4 θ sin 2 φ cos φ + sin 2 θ cos 2 φ cos φ ]dθdφ
S1

By integrate on φ firstly
π  2π
1 1 
∫∫S F .d s1 = a ∫0 sin θ cos θ . 2 θ + 2 sin 2φ 
4 2 3

1 

1 3   π
π
+ sin θ  sin φ  + sin θ cos θ [sin φ ] 0  dθ = πa ∫ sin 3 θ cos θdθ = a 4
4 2 2 2π 4 2
(1)
3 0 
0 4

On the surface S 2

ds 2 = − k ds ∴ F.ds 2 = − xz ds
But on the surface S 2 , z=0.

∫∫ F.ds
S
2 =0 (2)

From (1) , (2) we get

a 4π
∫∫ F .d s =
S
4
Second: by using Gauss theorem.

∫∫ F .d s = ∫∫∫ div F dV
S V

where
div F = z + 2 x = r cos θ + 2risnθ cos φ
dV = r 2 sin θdrdθdφ
div F dV = r 3 (sin θ cos θ + 2 sin 2 θ cos φ )drdθdφ

52
π
Where, 0 < r < a , 0<θ< , 0 < φ < 2π
2

( )
π
r 3 sin θ cosθ [φ ] 0 + 2 sin 2 θ [sin φ ] 0 drdθ
a
∫∫∫ div F dV = ∫ ∫ 2 2π 2π
0 0
V
π π
= 2π ∫
a

0 ∫ 0
2 r 3 sin θdθdr = 2π
a4 1
[
. sin 2 θ
4 2
] 2
0

πa 4 πa 4

∫∫∫
V
div F dV =
4
then ∫∫ F .d s =
S
4

Stokes’ Theorem :

Which states that:


The linear integral for the vector A around the closed curve C is equal to the
surface
Integral for (curl A) around any surface S bounded by the closed curve.
mathematically form: "

∫∫ (curl A).d s = ∫ A.d r


S C

Proof:
‫ وذﻟك �ﺎﺴتخدام ﻤنحن�ﺎت ﻤتواز�ﺔ �مﺎ ﻫـو ﻤبـین �ﺎﻟشـكﻞ‬S i ‫ إﻟﻰ ﻤجموﻋﺔ ﻤن اﻟسطوح‬S ‫ﻨﻔرض أﻨنﺎ ﻗسمنﺎ اﻟسطﺢ‬
‫ و�ﺎﻟتﺎﻟﻲ ﻓﺈن‬ABCD ‫ ﻤسﺎﺤتﻪ ﻫﻲ‬S1 ‫ﻓمثﻼ اﻟسطﺢ‬

∫ A.d r = ∑ ∫
C i
Ci
A.d r

‫وذﻟــك ﻷن‬ ∫ A.d r ‫ و�ت�ﻘــﻰ ﻟنــﺎ ﺘكﺎﻤــﻞ واﺤــد ﻓﻘــط ﻫــو‬S i ‫ اﻟــذي �حــد اﻟســطﺢ‬ABCDA ‫ ﻫــو اﻟمنحنــﻰ‬C i ‫ﺤیــث‬
GF

.(‫اﻟتكﺎﻤﻼت �ﻼﺸﻰ �ﻌضﻬﺎ )أي أﻨﻬﺎ ﺘتﻼﺸﻰ ﺠم�ﻌًﺎ إﻻ ﻋﻠﻰ ﺤﺎﻓﺔ اﻟسطﺢ‬
‫وﻋﻠ�ﻪ ﻓﺈن‬
∫∫ (∇ ∧ A).d s = ∑ ∫∫ (∇. A).d s
S i Si

:‫و�ﺎﺴتخدام ﺘﻌر�ف دوران اﻟمتجﻪ ﻨحصﻞ ﻋﻠﻰ‬


∫∫ (∇ ∧ A).d s = ∫ A.d r
S C

:‫وﻓﻰ اﻹﺤداﺜ�ﺎت اﻟكﺎرﺘیز�ﺔ ﻨجد أن ﻨظر�ﺔ ﺴتو�س ﺘﺄﺨذ اﻟصورة‬

53
 ∂A3 ∂A2 
∫ ( A dx + A dy + A dz ) = ∫∫  ∂y
C
1 2 3
S
−  cos α
∂z 
 ∂A ∂A   ∂A ∂A  
+  1 − 3  cos β +  2 − 1  cos γ  ds
 ∂z ∂x   ∂x ∂y  
. (xy ‫ ﻫﻲ زاو�ﺎ اﺘجﺎﻩ اﻟﻌمودي ﻋﻠﻰ اﻟسطﺢ وﻓﻰ ﺤﺎﻟﺔ اﻟمستوى ﺘﺄﺨذ اﻟصورة )اﻟمستوى‬α, β, γ ‫ﺤیث‬

 ∂A2 ∂A1 
∫ ( A dx + A dy) = ∫∫  ∂x
C
1 2
S
− dxdy
∂y 

‫ و�ـذﻟك‬xy ‫ ﺘﻘﻊ ﻋﻠﻰ اﻟمسـتوى‬S ‫ واﻟمسﺎﺤﺔ‬A ‫واﻟبرﻫﺎن ﯿنتﺞ ﻤ�ﺎﺸرة ﻤن ﻨظر�ﺔ ﺴتو�س اﻟﻌﺎﻤﺔ �ﺎﻋت�ﺎر أن اﻟمتجﻪ‬
π
A 3 = 0, cos α = cos β = cos = 1
2
Results:

(1) If the vector A is irrotational (curl A = 0) , then ∫ A.d r = 0 . i.e. the necessary
C

and sufficient condition that ∫ A, d r = 0 ,for all closed curve C


C
is ∇ ∧ A = 0 .

(2) If A = a ∧ b , where b is a constant vector ,then

 
∫∫ [∇ ∧ (a ∧ b)].d s = ∫ (a ∧ b ).d r ⇒
S C
b.∫∫ (d s ∧ ∇) ∧ a =  ∫ d r ∧ a .b
S C 
then
∫∫ (dS ∧ ∇) ∧ a = ∫ dr ∧ a
S C

⇒ ∫∫ ((n ∧ ∇) ∧ a)dS = ∫ dr ∧ a
S C

Solved problems:

(1) Verify Stokes theorem for a vector A where A = (2 x − y )i − yz 2 j − y 2 z k ,


and S is the half upper surface of a sphere x 2 + y 2 + z 2 = 1 .

Solution:
The bounded C for a surface S is a circle has a unit radius and its centre in xy
plane,
then
x = cos θ , y = sin θ , z = 0 , 0 ≤ θ ≤ 2π
∴∫ A.d r = ∫ (2 x − y )dx − yz 2 dy − y 2 zdz
c c

54

= ∫ (2 cos θ − sin θ )(− sin θ )dθ = π (i )
0

i j k
d d d
∇∧A= =k
dx dy dz
2 x − y − yz 2 − yz 2

then
1 1− x 2

∫ ∫ (∇n A).d S = ∫ ∫ k.n dS = ∫


s s
∫ dy dx
x = −1 y = − 1− x 2

Where
k .n dS = dxdy on the circle x 2 + y 2 = 1

1 1− x 2 1

∫ ∫ (∇n A).d S = 4 ∫ ∫ dy dx = = 4∫ 1 − x dx = π
2
(ii )
s x =0 y =0 0

(2) Verify Stokes’theorem for a vector A = z 2 j − y 2 k , S is the triangle


OAB which
O(0,0,0) , A(0,2,0) , B (0,0,3) , C is its boundary.

Solution:
It is clear that the given triangle lie in the plane x = 0
Then the line integral is ∫ A.d r ,where the closed curve C
c

Consists of the three paths C1 , C 2 , C 3 .

The integral

First: The line integral.


The line integral on C1 :
r ≡ r ( x, y , z ) , x = z = 0
A.d r = z 2 dy , z = 0 ∴ A.d r = 0
then ∫ A.d r = 0 (i )
c1

The integral on C 2
x = 0 ⇒ A.d r = z 2 dy − y 2 dz

55
y z
Where the equation of the straight line C 2 is + = 1 , then
2 3
2 3
y= (3 − z ) , z = (2 − y )
3 2
9
∴ A.d r = [(2 − y ) 2 dy − (3 − z ) 2 dz ]
4
0 3
9 4
∴ ∫ A.d r = [ ∫ (2 − y ) 2 dy ] − [ ∫ (3 − z ) 2 dz ]
|C 2
4 y =2 9 z =0

9 1 
0 3
4 1
= − (2 − y ) + . (3 − z )
3 3

4  3 2 9 3 0

3 4
= [−8] − [27]. = −6 − 4 = −10 (ii )
4 27

The integral on the path C 3 :


where x = y = 0 , then
A.d r = − y 2 dz ,y =0 ⇒ A.d r = 0
∴ ∫ A.d r = 0 (iii )
C3

Then the integral on the closed curve is ∫ A.d r = 0 + (−10) + 0 = −10


C

Second: To find the surface integral ∫ ∫ ∇ ∧ A.d s


s

where
i j k
d d d
∇∧ A= = −2( y + z )i
dx dy dz
0 z2 − y2
d s = nds = ids = idydz ,x = 0
2 3 / 2( 2− y )
 y 
2 3 2
( 2− y )
∴ ∫ ∫ (∇n A).d s = −2 ∫ ∫ ( y + z )dydz = − ∫ ( y + z ) 2 2 dy = − ∫ − y 2 + (3 − ) 2 dy
0 
s y =0 z =0 0
z =0 2 
 y2 
2
3 1 
2 2
3 3 2
= ∫  y 2 − (9 − 3 y + )dy = ∫ ( y 2 + 3 y − 9)dy =  . y 3 + y − 9 y 
0 
4  0
4 4 3 2 0
= (2 + 6 − 19) − (0) = −10

From first and second, Stoke’ theorem is verified.


∫∫ (∇ A).d s = ∫ A.d r
S C

Green’s Integrals Theorems:

56
∫∫∫[φ∇ ψ + ∇φ.∇ψ ]dV = ∫∫ (φ∇ψ ).d s
2
(1)
V s

Proof:
Let A = φ∇ψ in the Gauss’s divergent theorem
∫∫∫ (∇. A) = ∫∫ A.d s
V s
we get

∫∫∫ ∇.(φ∇ψ )dV = ∫∫ (φ∇ψ ).d s


V s

∴ ∫∫∫[φ∇.(∇ψ ) + ∇ψ .∇φ ]dV = ∫∫ (φ∇ψ ).d s


V s

∴ ∫∫∫[φ∇ 2ψ + ∇φ .∇ψ ]dV = ∫∫ (φ∇ψ ).d s (1)


V s

(2) ∫∫∫[φ∇ 2ψ − ψ∇ 2φ ]dV = ∫∫ [φ∇ψ − ψ∇φ ].d s


V s

Proof:
Interchanging φ & ψ , in Eq. (1)
∫∫∫[ψ∇ φ + ∇φ.∇ψ ]dV =∫∫ (φ∇ψ ).d s
2
(2 )
V s
Subtracting (2) from (1), we have
∫∫∫[φ∇ ψ −ψ∇ φ ]dV = ∫∫ [φ∇ψ −ψ∇φ ].d s
2 2
(3)
V s

(3) ∫∫∫ (∇ ∧ A)dV = ∫∫ dS ∧ A


V s

Proof:
Let B = A ∧ C in the Gauss divergent theorem, where C is a constant vector.
∫∫∫ (∇. B )dV = ∫∫ B.d S ,
V s

∫∫∫ ∇.( A ∧ C )dV =∫∫ ( A ∧ C ).d S


v s
(4)

 ∇.( A ∧ C ) = ( A ∧ C ).d S where ∇ ∧ C = 0


( A ∧ C ).d S = C.(d S ∧ A)
∴ ∫∫∫ C.(∇ ∧ A)dV = ∫∫ C.(d S ∧ A)
V s

∴ C.[ ∫∫∫ (∇ ∧ A)dV − ∫∫ C.(d S ∧ A)] = 0


V s

and since C is an arbitrary constant vector, we can write :


∫∫∫ (∇ ∧ A)dV − ∫∫ d S ∧ A = 0
V s

⇒ ∫∫∫ (∇ ∧ A)dV = ∫∫ d S ∧ A
V s
(5)

(4) ∫ φd r = ∫∫ d S ∧ ∇φ
c s

Proof

57
(4) ∫ φd r = ∫∫ d S ∧ ∇φ
c s

Let A = φ C
∫∫ (∇ ∧ A).d s = ∫ φ C.d r
s c

∇ ∧ φ C = ∇φ ∧ C sin ce ∇ ∧ C = 0
∴ ∫∫ (∇φ ∧ C ).d s = ∫ C.φd r
s c

∴ ∫∫ C.(d s ∧ ∇φ ) ) = ∫ C.φd r
c
s

∴ C.[ ∫∫ d s ∧ ∇φ − ∫ φ .d r ] = 0
s c

and since C is an arbitrary constant vector, we can write :


∫∫ d s ∧ ∇φ − ∫ φ.d r = 0
s c

⇒ ∫∫ d s ∧ ∇φ = ∫ φ .d r
s c

(5) ∫∫∫ ∇φdv = ∫∫ φd s


v s

Proof:
Let A = φ C , where C is a constant vector
From Gauss theorem, we have
∫∫∫ (∇. A)dv = ∫∫ (φ C ).d S
v s

∴ ∫∫∫ ∇.(φ C )dv = ∫∫ (φ C ).d s


v s

∴ ∇.(φ C ) = ∇φ .C because ∇.C = 0


∴ ∫∫∫ (∇φ .C )dv = ∫∫ C.φ .d s
v s

And since C is an arbitrary constant vector, then


∴ C.[ ∫∫∫ ∇φdv − ∫∫ φ .d s = 0
v s

∴ ∫∫∫ ∇φdv = ∫∫ φ .d s
v s

Exercises

(1) Prove that:


(i ) ∇ ∧ ( A + B) = ∇ ∧ A + ∇ ∧ B
(ii ) ∇ ∧ (φ A) = φ∇ ∧ A + ∇φ ∧ A
(iii ) ∇.( A ∧ B ) = B.(∇ ∧ A) − A.(∇ ∧ B )
(iv) ∇( A.B ) = ( B.∇) A + ( A.∇) B + B ∧ (∇ ∧ A) + A(∇ ∧ B)
(v) ∇ ∧ ( A ∧ B ) = ( B.∇) A − B(∇. A) − ( A.∇) B + A(∇.B)

58
(vi) ∇ ∧ (∇ ∧ A) = ∇(∇. A) − ∇ 2 A

(2) Prove that:

(i ) ∇ ∧ r = 0
(ii ) ∇ ∧ (r 2 C ) = −2C ∧ r
(iii ) ∇.(r 2 C ) = 2C.r
(iv) (C ∧ ∇) ∧ r = −2C
(3) If the vector A is solenoidal and irrotational vector, prove that for any constant
vector C
∇(C. A) + ∇ ∧ (C∇ A) = 0
(4) Prove that: ∇.( A∇ r ) = 0 , if ∇ ∧ A = 0 .
1
(5) If V = w ∧ r , prove that w = curl V , where w is a constant vector.
2
∂h ∂E
(6) If ∇.E = 0, ∇.H = 0, ∇ ∧ E = − , ∇ ∧ H =
∂t ∂t
∂ 2u
Prove that E, H satisfy the wave equation ∇ 2 u = 2 .
∂t
(7) If A, B are irrotational vectors, prove that ∇u ∧ ∇v is solenoidal.
(8) If A, B are irrotational vectors, prove that the vector A∇ B is solenoidal.
(9) Prove that the vector A = yzi + zx j + xy k is solenoidal and irrotational.
(10) If the vector A is irrotational but φ A is rotational where φ ( x, y, z ) is a scalar
function
Prove that A.∇ ∧ (φ A) = 0 .
(11) If S is an open surface bounded by a closed curve C . If A is a constant vector
field r is the position vector w.r.t. the origin. From Stokes theorem prove that
1
∫c ( A ∧ r ).dr = 2∫∫S A.dS , then prove that ∫∫s ds = 2 C∫ r ∧ dn .
(12) If S is an open surface bounded by a closed curve C , and φ & ψ are scalar
functions. By using Stokes theorem prove that ∫ φ∇ψ . dr = ∫∫ ∇φ ∧ ∇ψ . dS .
C S

(13) Verify Stokes theorem for a vector field A = − y i + x j on the path C , where C
is a circle in xy plane which center is the origin and its diameter is a.
∫∫ ∇ ∧ F . dS = 6πa , where S is x 2 + y 2 − 2ax + az = 0 & x ≥ 0 , and the
3
(14) Prove that
S

Vector field is F = (2 y 2 + 3z 2 − x 2 ) i + (2 z 2 + 3x 2 − y 2 ) j + (2 x 2 + 3 y 2 − z 2 ) k .
(15) By using Stokes theorem, Find ∫∫ curl A . dS around the circle

( x − a ) 2 + ( y − b) 2 = c 2 in the plane z = 0 , if A = − y 2 i + cx j .
(16)(a) Change the volume integral ∫∫∫ ∇φ . (∇ . A) dV to surface integral.
V

59
(b) Change the closed line integral ∫ φ dψ to surface integral.
C

(c) Prove that


∫∫ (B ∧ curl A − A ∧ curl B ). dS = ∫∫∫ ( A . curl curl B − B . curl curl A) dV .
S V

(17) If S is the surface of the sphere r = (a sin θ cos φ , a sin θ sin φ , a cos θ ) , where
0 ≤ φ ≤ 2π , 0 ≤ θ ≤ π . Prove that ∫∫ (x )
+ y 2 dS = 0 .
2

(18) Calculate the surface integral ∫∫ r dS , where S is the surface of the half sphere
S

x + y + z = a & z ≥ 0 , but r is the position vector w. r.t . the origin.


2 2 2 2

(0,0, πa ).
3

(19) Verify divergence Gauss theorem, where A = x 3 i − y 3 j + xz 3 k and S is the


surface of the cylinder which its axes is z- axis and its radius a and lie between
the planes z = 0 & z = h .
(20) By using Gauss theorem evaluate the surface integral of A = xe z i + y j + z k
around the closed surface containing the cylinder which its center is
(0,0,1) , its axis is z- axes, its radius a and its height is 2.
(21) Verify Gauss theorem if A = 3x i + ( y − 3) j + (2 − z ) k and S is the closed surface
of closed box which sides are the planes
x = 0 & x =1 & y = 0 & y = 2 & z = 0 & z = 3
16 3
(22) Verify Gauss theorem if A = x i , and the surface is a cube which side is 2
3
and its center is the origin.
(23) By using Gauss theorem, evaluate the surface integral of A = 4 x 2 i − 2 y 2 j + z 2 k ,
on the cylinder surface x 2 + y 2 = 4 & z = 0 & z = 3 .

60
Chapter III

Field and Potential Theory

First: Conservatives Fields.


The vector field F is said to be conservative field if the line integral of this vector
Q
between the points P, Q , i.e. ∫ P
F .d r , independent of the path but depend on the end of
the path.
Suppose that F = φ (r )e r , then the line integral for the vector F between P, Q is

Q ( r2 ) Q r.d r
∫ P ( r1 )
F .d r = ∫ φ (r )
P r
r
Where e = ,
r
But r.d r = rdr ⇐ r.r = r 2
Q Q
∴ ∫ F .d r = ∫ φ (r )dr = ψ (r ) |QP = ψ (r2 ) − ψ (r1 )
P P


where = φ (r )
dr
Theorem (1):
If the vector field F is conservative then its line integral around any closed curve equal
zero, i.e.
∫ F.dr = 0 .
c
We can prove that from the previous definition, where the conservative field independent
of the path but depend on ends of the path, then

P
∫ F .d r = ∫
c
P
F .d r = ψ ( p ) − ψ ( p ) = 0

Theorem (2):
The necessary and sufficient condition that the vector field F is conservative field
is F = −∇φ , where φ is the scalar function.
Proof:
(i) to prove the necessary condition, suppose that F is a conservative field , then
the line integral between the points P( x, y, z ), Q( x1 , y1 , z1 ) independent of the path but
depend on P, Q. i.e.

Q ( x1 , y1 , z1 )
∫ P ( x, y, z )
F .d r = −φ ( x, y, z ) + φ ( x1 , y1 , z1 )
= φ ( x1 , y1 , z1 ) − φ ( x, y, z )
Q Q dr
∴ ∫ F .d r = ∫ F . = φ ( x1 , y1 , z1 ) − φ ( x, y, z )
P P d

61
By differentiation w.r.t.

dr dφ ( x, y, z ) dφ dr
F. =− but dφ = ∇φ .d r ⇒ = ∇φ .
d d d d
Then
dr dr dr
F.= −∇φ . ⇒ ( F + ∇φ ). =0
d d d ֶ
⇒ F = −∇φ
(ii) To prove the sufficint condition, suppose that φ is the scalar function such that

F = −∇φ .

P P P
∫ F .d r = ∫ F .d r = −∫ ∇φ.d r − ∫ dφ = φ ( p) − φ ( p) = 0
C
P P P

From (i), (ii) we get F = −∇φ , is a conservative field . φ called Potential.

Theorem (3):

The necessary and sufficient condition that the field F is conservative is ∇ ∧ F = 0


Proof:
From Stokes theorem ∫∫ (∇ ∧ F ). dS = ∫ F .d r
S C

If the field is conservative then from theorem (1) for any closed curve C around any
point is
∫ F.dr = 0 , then ∫∫ (∇ ∧ F ) . dS = 0 .
C
S

But S is an arbitrary surface the point , then ∇ ∧ F = 0 , at any point


reversly,
If ∇ ∧ F = 0 at any point then ∫∫ ∇ ∧ F . dS = 0 .
S

Ex) Show that a necessary an sufficient condition that F1 dx + F2 dy + F3 dz be an exact


differential is that ∇ ∧ F = 0 , where F = F1 i + F2 j + F3 k
Proof:
∂ψ ∂ψ ∂ψ
Suppose F1 dx + F2 dy + F3 dz = dψ = dx + dy + dz
∂x ∂y ∂z
An exact differential then
∂ψ ∂ψ ∂ψ
F1 = , F2 = , F3 =
∂x ∂y ∂z
∂ψ ∂ψ ∂ψ
∴ F = F1 i + F2 j + F3 k = i+ j+ k
∂x ∂y ∂z
where ψ potential function , so F = ∇ψ .Then ∇ ∧ F = ∇ ∧ ∇ψ = 0
Conversely, If ∇ ∧ F = 0 then is conservation field and F = ∇ψ
∴ F .d r = ∇ψ .d r = dψ which is an differential.

62
Summary:
If F = −∇φ in any region R defined as c1 ≤ u 3 ≤ c 2 , a1 ≤ u1 ≤ a2 , b1 ≤ u 2 ≤ b2 , where
φ (u1 , u 2 , u3 ) has unique value and conteneous derivative in the region R, then
Q ( r2 )
(1) ∫ F .d r independent of the path C in the region R
P ( r1 )
between P , Q.

(2) ∫ F .d r = 0 around any closed curve C in R.


C

The vector field F is called conservative field iff F = −∇φ , ∇ ∧ F = 0 . But φ is potential.

Solved problems:
(1) If the vector field F = (2 xy + z 3 )i + x 2 j + 3 xz 2 k .
(i) Prove that the field F is conservative field.
(ii) Find the scalar potential φ.
(iii) Find the work done in moving an object in this field from P(1,−2,1) to Q(3,1,4).
Solution:
(i) The necessary and sufficient condition that the vector field is conservative is
∇∧F=0 .

i j k
∂ ∂ ∂
∇∧F = =0
∂x ∂y ∂z
2 xy + z 3 x2 3 xz 2
(ii) To find the potential φ we know that F = −∇φ , i.e.

∂φ ∂φ ∂φ
= −(2 xy + z 3 ), = −x2 , = −3 xz 2
∂x ∂y ∂z
φ = − x 2 y − xz 3 + f1 ( y, z ), φ = − x 2 y + f 2 ( x, z ), φ = − xz 3 + f 3 ( x, y )
we chose f1 ( y, z ) = 0, f 2 ( x, z ) = − xz 3 , f 3 ( x, y ) = − x 2 y
then
φ ( x, y, z ) = − x 2 y − xz 3 + const
Q
(iii) The work done ∫ P
F .d r is

F .d r = − ∫ ∇φ .d r = − ∫ dφ = [− φ ] P = φ ( P) − φ (Q)
Q Q Q
∫ F .d r = ∫
Q
P P P
c

∫ F .d r = φ (1,−2,1) − φ (3,1,4) = 202


C

(2) Prove that the field F = r 2 r Is conservative and find the potential φ.
Solution:

The condition of the conservative field is ∇ ∧ F = 0 , then

63
∇ ∧ F = ∇ ∧ (r 2 r ) = r 2 (∇ ∧ r ) + ∇r 2 ∧ r
= r 2 (0) = 2r ∧ r = 0 + 0 = 0
We have F = −∇φ , then
∂φ ∂φ r4
r er = − er + 0
3
∴ = − r ⇒ φ = − + const
3

∂r ∂r 4
(3) Prove that the vector field F = x y z ( x i + y j + z k )
m m m n n n

. n = −1 , m = 0 ‫( اﺜبت أن اﻟمجﺎل اﻻﺘجﺎﻫﻲ ﻏیر ﻤحﺎﻓظ إﻻ إذا �ﺎﻨت‬3)

Solution:
∇∧F =0 F is a conservative field
i j k
∇∧F =

∂x

∂y

∂z
[
= i mx m y m −1 z m z n − mx m y m z m −1 z n ]
m m m n m m m n
x y z x x y z y xm ym zm zn
[ ] [
+ j mx n y m z m −1 x n − mx m −1 y m z m z n + k mx m −1 y m z m y n − mx m y m −1 z m x n ]

= m[i( x m y m −1 z m + n − x m y m + n z n −1 ) + j ( x m + n y m z n −1 − x m −1 y m z m + n ) + k ( x m −1 y m + n z m − x m + n y m −1 z m )`
‫ ∧ ∇ وﻫذا ﻻ ﯿتحﻘق إﻻ إذا �ﺎﻨت‬F = 0 ‫ ﻤجﺎل ﻤحﺎﻓظ ﻻﺒد أن �كون‬F ‫ﺤتﻰ �كون‬m= 0
Or m + n = m − 1 ⇒ n = −1
. m = 0 or n = −1 ‫ ﻏیر ﻤحﺎﻓظ إﻻ إذا �ﺎﻨت‬F ‫∴اﻟمجﺎل اﻻﺘجﺎﻫﻲ‬

Exercises
(1) If the vector field is
F (x, y, z ) = ( y 2 cos x + z 3 )i + (2 y sin x − 4 ) j + (3 xz 2 + 2 )k
(a) Prove that the field F is conservative.
(b) Find the potential φ .
π 
(c) Find the work done in moving an object from the point P(0,1,−1) to Q ,−1,2  .
2 
r
(2) Prove that the vector field 3 is conservative field and find potential function.
r
(3) Find the parameters α , β , γ which make the vector field F is conservative,

and then find ( ) (


F (x, y, z ) = y 2 z 3 cos x + α x 3 z i + β z 3 y sin x j + γ y 2 z 2 sin x − x 4 k )
potential function φ which related to the vector field F .

64
(4) If the vector field F is given by

( ) ( )
F (x, y, z ) = 2 y 2 z − 3 z 2 i + (4 xyz + 5) j + 2 xy 2 − 6 xz k
Prove that the field F is conservative and find potential φ .

(5) If a is constant vector, prove that the vector field (a. r ) r is not conservative,
but
the vector field (a. r ) a is conservative and find potential function.

Second: Solid Angle:


Let be a point on the surface the element of area at is . If the points on the
perimeter of is connected to the origin , we have a cone with its vertex at It follows
that the angle is the confined angle between and the unit to the surface at the
cos θdS
point If OP = r , we define dω = as the solid angle which corresponding to the
r2
element of area at 0, such that
dΩ
dω = 2 : dΩ = cos θdS
r
It follows that the total solid angle that corresponding to the total surface S is given by
cos θdS r.d S r.n
ω = ∫∫ 2
= ∫∫ 3 = ∫∫ 3 dS
S r S r r
Therefore, if is an acute angle, the solid angle that corresponding to the surface at is
equal to the cutting area from the cone with the unit circle at Meanwhile, if is an
obtuse angle, the solid angle that corresponding to the surface at equals to (-1) is
multiple of the cutting area from the cone with the unit circle at 0.
Finding the solid angle in some special cases:
First if the point is outside the closed surface
From Gauss’ divergent theorem, we have
r  r 
∫∫ r
S
3
d S = ∫∫∫ ∇. 3  dV
V r 
 r 
Where is the volume surrounds the surface . But we have ∇. 3  = 0 at any point
r 
 r 
inside the volume , if r ≠ 0 inside , it follows that we have ∇. 3  = 0 . Therefore, the
r 

65
total solid angle the corresponding to the closed surface s at any point 0 outside the closed
surface is zero.
Second: If the point is inside the closed surface
In this case, we surround the point 0 by a sphere of radius and its surface such that it
lies entirely inside the surface . Let be the region surrounds by and , from Gauss’
theorem, one gets
r r r r
∫∫ r
S +S '
3
.d S = ∫∫
S r 3
d S + ∫∫ 3 .d S = ∫∫∫ ∇. 3 dV
S1 r V r
Since r ≠ 0 inside , we found

r r.n
ω = ∫∫ 3
d S = − ∫∫ 3 dS
S r S' r

Since on n = −e r , r = ae r
r.n 1
∴ 3
=− 2
r a
Hence, we have
r 1 1
ω = ∫∫ 3
.d S = 2 ∫∫ dS = a 2
.4πa 2 = 4π
S r a S'

Therefore, the solid angle at a point inside a closed surface is equal to 4π. If the point lies
outside the surface, the solid angle equals zero.

Third: Attraction and potential


1. Newton’s law of gravitation
States that: each particle attracts another particle by an attraction force which is direct
proportional with their masses product and inversely proportional to the square distance
between them.
Therefore, if we have m1 , m 2 are the masses of two particles at distance apart, the
magnitude of the attraction force is given by
m1m2
, F =γ
r2
where is the proportional constant. In C.G.S. system it is equal 6.66 × 10 −8
Therefore, if we have a particle of mass is located at a point , and a unit mass is
placed at point at distance from , we have
γm
F =− 3 r
r

66
The negative sign indicates that the attraction force F is directed from to , while is
directed from to . Clearly, the attraction force F is a conservative force, since

 r 
[
∇ ∧ F = −γm∇ ∧  3  = −γm r −3∇ ∧ r + (∇r −3 ∧ r ) = 0 ]
r 
Therefore, the potential of this field is given by
F = −∇φ
r r r.d r r dr γm
φ = − ∫ F .d r = γm ∫ 3 = γm ∫ 2 = −
∞ ∞ r ∞ r r
2. Gauss’s Theorem for attraction
If we have some particles of masses m1 , m2 , m3 ,.... , the flux of the resulted attraction force
in accordance of a closed surface is given by ∫∫ F .d S = −4πγM , where
S
represents the

total masses inside the closed surface


Proof:
Suppose that the masses m1 , m2 , m3 ,.... have the position vectors r 1 , r 2 ,... . Let N1 be the
flux of attraction force due to the mass m1 across the closed surface S, therefore, we have
γm1 r 1 r 1 .d S
N 1 = ∫∫ F 1 .d S = − ∫∫ 3
.d S = −γm1 ∫∫ = −γm1 ∫∫ dω1
S S r 1 S r13 S

If ω1 is the solid angle corresponding to the particle m1 , and the point lies outside the
closed surface , the solid angle becomes zero. Meanwhile, if the point lies inside the
solid angle becomes 4π. Therefore, we have
N1 = −γm1 (4π ) = −4πγm1
The total flux for all the particles
N = ∑ N i = ∑ − 4πγmi = −4πγ ∑ mi = −4πγM
i i i

where represents the total masses inside the closed surface


Hence, we have

∫∫ F .d S = −4πγM
S

3. Poisson’s Equation
If F is the attraction force intensity, obeying the Newton’s gravitation, at the point P, we
have div F = −4πγρ , where is the mass density at the point
Proof:
Consider the mass density at any is . Draw an arbitrary closed surface , surrounds a
volume . From Gauss’ attraction theorem, we have

67
∫∫ F .d S = −4πγM (1)
∆S

But, we have M is the total mass inside , we have


M = ∫∫∫ ρ dV (2)
∆V

From Gauss’ theorem on the closed surface

∫∫ F .d S = ∫∫∫ div F dV (3)


∆S ∆V

From (1), (2) and (3), one gets

∫∫∫ div F dV = −4πγ ∫∫∫ ρ dV


∆V ∆V

Then ∫∫∫[div F + 4πγρ ]dV = 0


∆V

Since is an arbitrary volume, it follows that the integrand must vanishes, Therefore, we
have
div F + 4πγρ = 0
Or div F = −4πγρ
Which is a partial differential satisfying the attraction force F . If we have , one
gets
div F = 0 , which is called Laplace’s equation.
Potential:
The potential of the attraction force F at any point is defined as the work done from this
force on a unit mass from infinity to the point
To find the potential for a particle of mass at the point K ( x 1 , y1 , z1 ) , we have
K ( x1 , y1 , z1 ) γm
φ=∫ F .d r : F =−
∞ r3
1 1
K
r.d r K K dr 1 
∴φ = −γm ∫ = −γm ∫ r 3 = γm r  ∞ = γm rK − r∞ 
∞ r3

 γm 
Therefore, the potential of mass at any point at distance r apart is  φ =  , which is a
 r 
scalar quantity.
1. For a set of discrete distribution of particles of masses m1 , m 2 ,... , the resulted
scalar potential is given as
 m1 m2 
φ = γ  + + .... 
 r1 r2 
2. For a continuous distribution, the scalar potential is given as
dm
φ =γ∫
r

68
Corollary
The work done to transport a unit mass in accordance of the attraction force F from the
point to a point is given by [φ ( K ) − φ (Q)].

The relation between attraction and potential


The relation between attraction and potential is
F = −∇φ
Proof:
Let and are two adjacent points on a curve, the work done to move a unit mass from
the point to the point is the difference in the potential functions between these two
points.
∴ F .d r = φ ( K ) − φ (Q)
= φ ( K ) − {φ ( K ) + dφ } = −dφ
But dφ = ∇φ.d r Then F.d r = −∇φ.d r (F + ∇φ ).d r = 0
Since we have an arbitrary vector, it follows that
F + ∇φ = 0
Therefore, we have
F = −∇φ
Poisson’s potential equation
From the Poisson’s attraction equation, we have
div F = −4πγρ
Since F = − grad φ , therefore, − div γρadφ = −4πγρ
Then ∇.∇φ = 4πγρ ⇒ ∇ 2φ = 4πγρ
This is the scalar form for Poisson’s equation at any point in space. If , at a given
point, it follows that the potential satisfies the Laplace’s equation
∇ 2φ = 0
In this case, is called a harmonic function.

First: The boundary conditions that must be satisfied by the attraction


force at the surface of separation
At the surface of separation , between two media, we must have
1. The tangential components of attraction force must be continuous.

69
2. The normal components of attraction force must discontinuous by the amount
− 4πγδ , where be the surface density of the material at the surface of separation
.
Proof:
1. Consider the surface of separation separates two media as shown in the figure, let
and are two adjacent points on .
Draw A1 B1 , A2 B2 parallel to in the two different sides through the regions (1), (2).
Suppose that the tangential components of the field in these two regions are Ft1 and Ft2
and the distance A1 A2 , B1 B2 is infinitesimal.
Since the field F is conservative, we have

∫ F .d r = 0
C
⇒ ∫ A1 B1 B2 A2
F .d r = 0

Then
B1 B2 A2 A1
∫ A1
F .d r + ∫
B1
F .d r + ∫ F .d r + ∫ F .d r = 0
B2 A2
B2 A1
∴ Ft1 ( A1 B1 ) + ∫ F .d r − Ft2 ( A2 B2 ) + ∫ F .d r = 0
B1 A2

B1 , B 2 → B; A1 , A 2 → A
Then
B2 A2
∫ B1
F .d r = 0 = ∫ F .d r ,
A1
A1 B1 = AB = A2 B2

Therefore, one gets


Ft1 ( AB) − Ft2 ( AB) = 0
Since , then Ft = Ft
1 2

It follows that the tangential components of attraction force must be continuous.


2. Consider an infinitesimal element of area on the surface of separation at the point
P. From the points of the perimeter on S.
Draw stream lines for the conservative field F in the two different sides through the regions
(1), (2). The generators of the cylinder S1S 2 are along to the direction of the field.
Suppose that the surface density of the material on the surface is , from Gauss’
theorem, we have

∫∫ F .d S = −4πγM (1)
S

Where is the total mass inside the cylinder.


Since the generators of the cylinder are parallel to the field, we have

∫∫ F .d S = ∫∫ ∇.d S + ∫∫ F .d S + ∫∫ F .d S = 0 ,
S S1 S2 S3

Where S3 is the lateral surface of the cylinder.


‫ ﻫو اﻟسطﺢ اﻟجﺎﻨبﻲ ﻟﻼﺴطواﻨﺔ‬S3 ‫ﺤیث‬

70
Then ∫∫ F .d S = − ∫∫ F n1 dS1 + ∫∫ Fn2 dS 2 = Fn2 S 2 − Fn1 S1 ,
S S1 S2

where Fn is the normal component of the field across the surface.


Finally, when S1 , S 2 is coincident on , we have

‫ ﻓﺈن‬S ‫ ﻋﻠﻰ‬S1 , S 2 ‫ ﻫﻲ ﻤر��ﺔ اﻟمجﺎل اﻟﻌمود�ﺔ ﻋﻠﻰ اﻟسطﺢ وﻓﻲ اﻟنﻬﺎ�ﺔ ﻋندﻤﺎ ﯿنطبق �ﻞ ﻤن‬Fn ‫ﺤیث‬

∫∫ F .d S = S ( F n2 − Fn1 ) (2)
S

The total mass inside the cylinder is which is equal to , from (1) and (2), we get
S ( Fn2 − Fn1 ) = −4πγδ S
∴ Fn2 − Fn1 = −4πγδ
Therefore, the normal components of attraction force must discontinuous by the amount
− 4πγδ
Second: The boundary conditions that must be satisfied by the potential
at the surface of separation
At the surface of separation , between two media, we must have
1. The potential is a continuous function, i. e. (φ1 = φ 2 )
 ∂φ ∂φ 
2.  1 − 1 = −4πγδ 
 ∂n ∂n 
Proof:
∂φ
1. Since Ft1 = Ft 2 , and F = −∇φ ⇒ Ft = − ,  is the arc length, then
∂
∂φ1 ∂φ 2
= ⇒ (φ1 = φ 2 )
∂ ∂
∂φ
2. Since Fn 2 − Fn1 = −4πγδ . Also, Fn = − , where n is the unit normal to , then
∂n
∂φ1 ∂φ 2
− = −4πγδ
∂n ∂n

Solved Examples
1. The attraction field and its potential for an infinite thin rod at any point outside
it.
A. Since the rod is an infinite, consider the required point is located at . Draw a
cylinder, such that its axis is the rod itself. Also, the point P lies on the surface of
the cylinder. From the symmetry, the direction of the attraction force must be normal
to the surface of the cylinder. Therefore, from Gauss’ theorem, we have

71
∫∫ F .d S = −4πγM ,
S

where is the surface of the cylinder.


The integral vanishes on the two bases on the cylinder. It is existing only on the lateral
surface and equals (−2πrF) , where is the length of the cylinder and is its radius of the
cross section. Since M = ρS , S is the area of the cross section, we have
− 2πrF = −4πrS
2γrS
∴F =
r
Therefore, the attraction force of an infinite rod at a point outside it is inversely proportional
to the distance from the rod.
2γrS dφ
B. Since F = and F = −∇φ , in the scalar form F = − , we have
r dr
dφ 2γrS
=− , by the separation of variables, one gets φ = −2γrS ln r + c
dr r
Therefore, the potential at the point P is given by
φ = −2γrS ln r + c ,
where is the constant of integration.

2. The attraction field and its potential of a uniform spherical shell at any point:
A. Outside the sphere B. Inside the sphere
For the attraction force
First:
Since the sphere is a uniform, from the symmetry, the attraction force at any point must
be directed along the radius, of the sphere and toward to its center . Since .
From Gauss; theorem, we have

∫∫ F .d S = −4πγ M
S

where is the mass inside , Spherical shell’ mass, since has the same value and is
directed along the radius, we have

∫∫ F .d S = ∫∫ FdS = F (4πa
2
)
S S

∴ 4πa F = −4πγ M
2

γM
∴F = − 2
a
Therefore, the attraction force due to a spherical shell at any point outside it equals the
attraction force due to a point of the same mass of the spherical shell and located at its
center.

72
Second:
If the point is located inside the spherical shell, i. e. , then the inner surface of will
lie inside the shell, and then ∫∫ F .d S = −4πγM , since
S
contains no material, it follows that

. Hence, we have ∫∫ F .d S = 0 . Therefore,


S
F = 0.

Finally, the attraction force according to the spherical shell may be written as
 γM
− :a>r
F =  a2
0 :a < r
For the potential function
γM
A. If lies outside the shell, then , and φ = − , which is the potential of a
a
point of mass M and located at the center of the shell.
γM
B. If lies inside the shell, then , and φ = const = − , where is the radius of
r
the shell, which shows that the potential at any point inside the shell is constant.
Notice that φ1 and φ 2 have the same value at the surface of separation r = a .
3. Prove that the potential of a rigid sphere of radius a and density at a point at
2
distance r from its center is equal to πγr(r 2 − 3a 2 ) , if . Hence, find the
3
potential function if . Also, the attraction force in the two cases.
Answer:
First for the attraction force
(i) If , inside the sphere, we have ∫∫ F .d S = −4πγ M
S

∴ F ∫∫ dS = −4πγ m
S

M  γ Mr
∴ F (4π r 2 ) = −4πφ  3 r 3  ∴F = −
a  a3
(ii) If , outside the sphere, we have
F (4π r 2 ) = −4πγ M
γM
∴F = −
r2
Therefore, we have
 γM
 − :r≥a
 r2
F =
− γM r = − 3 πγrr :r > a

 a3 4

73
where is the mass density of the sphere.
Second for the potential function
Since we have F = −∇φ
(i) The potential outside the sphere, i. e. , one gets
r rγ M γM
f = − ∫ fdr = ∫ 2 dr = −
∞ ∞ r r
Therefore, the attraction and potential for a rigid sphere are the same for a point of mass
equals to mass of the rigid sphere and located at its center.
(ii) The potential outside the sphere, i. e. , one gets
a r a γM r γM
φ = − ∫ Fdr − ∫ Fdr = ∫ 2 dr + ∫ 3 r dr
∞ a ∞ r a a

γM γM 2
=− + 3 (r − a 2 )
a 2a
2a γM γMr 2 γMa 2 γM 2
2
=− 3
+ 3
− 3
= 3 (r − 3a 2 )
2a 2a 2a 2a
2 4
∴ φ = πγρ (r 2 − 3a 2 ) : M = πρa 3
3 3
 γM
− r :r ≥ a
⇒φ = 
 2 πγρ (r 2 − 3a 2 ) :r < a
 3
4. Obtain the attraction force and its potential function for a rigid shell at a
point at distance r from its center.
Answer:
From the spherical symmetry of the shell, we found that the attraction force must has a
spherical symmetry also. Therefore, the field at any point in the space is constant if we
rotate the axes about its center. This means that the field and its potential must be a
function of the distance between the center and any point at which we want to find the field
and potential about it. Therefore, the field and its potential for the sphere is always
constant.
Now, we consider the following three cases:
(i) Case one, when
Surround the spherical shell with a spherical surface or radius , this called Gauss’ surface,
the field its always perpendicular to it. According to the Gauss’ theorem, we have

∫∫ F .d S = −4πγM ,
S

where is the mass of the spherical shell inside the surface. Since F has the same
value and always directed along the center, we have

74
∫∫ F .d S = ∫∫ FdS = F ∫∫ dS
S S S

γM
∴ F (4πr 2 ) = −4πγ ∴F = −
r2
(ii) Case two, when
4
In this case, the spherical surface of radius contains a mass of M ' = πσ(r 3 − a 3 ) , it
3
M(r 3 − a 3 )
may be written as , where M is the mass of the spherical shell.
(b 3 − a 3 )
Inside the shell, we have ∫∫ F .dS = −4πγM ' . According to Gauss’ theorem, one finds
S

4πγM (r 3 − a 3 ) γM (r 3 − a 3 ) 1
F (4πr 2 ) = − ∴F = −
(b 3 − a 3 ) (b 3 − a 3 ) r 2
(iii) Case three, when
In this case, Gauss’ surface contains no material, then ∫∫ F .d S = 0
S


The potential functions in the above cases may be obtained from the integration
φ = − ∫ F .d r
(i) When , we have
γM
r
r dr r  1
φ = − ∫ F .d r = γM ∫ 2 = γ M  −  = −
∞ ∞ r
 r∞ r
(ii) When , we have
  a3  
  r − 2  
 b dr r  
φ = − ∫ + ∫  F .d r = γM  ∫ 2 + ∫  3
b r r
dr 
 ∞ b  ∞ r b b − a3
 
 
 
 1 1  1 2 1 2 a 3 a 3 
= γM − + 3  r − b +
3 
− 
 b b −a 2 2 r b 
4 πγσ (b 3 − a 3 )  1 3  1 2 1 2 a 3 a 3  4  3 2 1 2 a3 
=  − (b − a 3
) + 
 r − b + −   = πγσ − b + r + 
3 b3 − a3  b 2 2 r b  3  2 2 r 
− 2πγδ  2 1 3 2 3 
∴φ =  rb − r − a 
r  3 3 
(iii) When , we have

75
φ = −  ∫ + ∫ + ∫  F .d r
b a r

 ∞ b a 
 b dr 1 a  a3  r  −3 (b 2 − a 2 )
= γM  ∫ 2 + 3 ∫b  r 2  ∫a  2 (b 3 − a 3 )
 r −  dr + 0 dr = γM
 r
∞ b − a3 
4
= −2πγd (b 2 − a 2 ) : M = πσ (b 3 − a 3 )
3
Some Special cases:
(1) As , we get the case of the rigid sphere, as follows
 γM  γM
− r 2 −
 r
, r≥b
F = , φ=
− γM  2 πγδ (r 2 − 3b 2 ), r < b
r
 b 3  3
(2) As , such that σ' = (b − a )σ which is the surface density of the shell, one
finds the field and its potential of a thin spherical shell as
 γM
 γM − r , r>R
− 2
F = r , φ=
0 − γM , r<R
 R
The last two cases are obtained earlier through examples (2) and (3).
If the relation between and are drawn, we notice that is a continuous
function at . Meanwhile, the function is discontinuous function at the same point. It
γm
suffers a finite discontinuity of value . This discontinuity is resulted according to the
r2
surface distribution on the spherical shell. It left to student to draw these two curves for the
rigid sphere. He will see that these two curves are continuous in the latter case.
5. Obtain the field according to a plate at any point normal to its plane.
Answer:
Consider a small element of area , the resulted field at any point outside it is given by
dS r
d F = −γσ 2 . .
r r
The total normal component is given as
dS
Fn = −γσ ∫ 3 n . r
r
d S .r
Since n is normal to the plane of the plate, we have n dS = d S , but dω = is the
r3
element of the solid angle according to this surface area at
∴ Fn = −γσω ,
where is the solid angle which corresponding to the plate at the point

76
Special Cases:
• Infinite plane plate
Firstly, we must find the solid angle, in this case, let the plate lies in the plane, and the
point P lies on the axis.
d S .r ∞ ( dxdy ) k .( xi + y j + z k )
∴ω = ∫ = ∫− ∞ ( x 2 + y 2 + z 2 ) 3 / 2
r3
∞ ∞ dx
ω = z ∫ dy ∫
−∞ −∞
(x + y 2 + z 2 )3
2

Consider the substitution x = ( y 2 + z 2 ) tan 2 θ , then x = ( y 2 + z 2 ) tan 2 θ .


It follows that
π
∞ dy
ω = z∫
− ∞ y 2 + z 2 ∫− π
2 cos θdθ

2
∞ dy
= 2z∫ = 2π
−∞ y 2 + z 2

If the point lies in the negative z-direction, we replace by . The solid angle in this
case becomes . Then the normal field in this case lies along positive direction,
and
Fn = −γσω = −2γσ
If lies on the negative z-direction, we have
Fn = −γσ (−2π ) = 2πγσ
Therefore, the normal component of the field is discontinuous. This result may be applying if
the point is very near to the plate. If very near to the plate, we can consider the plate has
infinite dimensions with relative to the distance from the point . From the forgoing, we
reach to the important result: If we have a continuous distribution on a plate surface, the
field suffers a finite discontinuity of value (−4πγσ) in the normal direction. In this case, we
have ( F 1 − F 2 ) . n = −4πγσ .
• Infinite strip
(i) Fn = −2αγσ
To find the solid angle when lies on normal line that dissects the strip, replace the strip by
some portion from a cylinder of cross section a 2 + b 2 as shown in figure

77

α
ρdφdz e ρ .( ρ e ρ + z k )
ω=∫ ∫α 2
−∞ −
2 (ρ 2 + z 2 )3
α
∞ ρ 2 dφ dz
=∫ ∫α 2
−∞ −
2 (ρ 2 + z 2 )3
∞ ρ 2 dz ρ 3 sec 2 θdθ
=α∫ =α∫ = 2α
−∞
(ρ 2 + z 2 )3 ρ 3 sec 3 θ
(ii) Let z = ρ tan θ
The normal component to the thin strip is given by Fn = −2αγσ , where is the even angle
a a
between the two planes passing the lines confining the strip. In this case tan = , from
2 b
Maclurin’s theorem, we have
tan −1 x = x − 4 x 3 + ....(−1) m (2m + 2) x 2 m+1
Since is very small, tan −1 x ~ x , but
a  a a
Fn = −4γσ tan −1   = −2γσ  2 tan −1  ~ − 4γσ
b  b b
− 2 γλ
Fn =
b
If is very small, we reach to the case of a rod. Notice that as is equivalent to the
case of the semi ring with radius at its center in the direction of the symmetry of the semi
ring.
• Rectangle
The required is the normal field to the rectangular plate at the point , where is the origin
in the cylindrical coordinates. The solid angle that corresponding is the same for the
spherical shell confining by the same rectangle.
φ2 rdθ , r sin θdφ
θ2
ω=∫ ∫θ
φ1 1 r2
= (φ 2 − φ1 (cosθ1 − cosθ 2 )
Fn = −γs (φ 2 − φ1 )(cosθ1 − cosθ 2 )
• If we consider θ1 = 0 and θ 2 = π , we obtain the case of the infinite strip
• If we take the case when P lies vertically as shown in figure, in this case, we have
α π π
Fn = −2γsα sin θ , φ 2 = −φ1 = and θ1 = − θ, θ 2 = + θ
2 2 2

78
π
• If we have θ → , we consider again the case of thin strip. Meanwhile, if the strip
2
a a
is thin and consider the approximation tan   = , where is the height of the
2 L
λ − 2γλb
point . As , one gets Fn = −2 γ sin θ , or Fn = .
L L L2 + b 2
(4) Circular disc at a point on its axis
For the symmetry, the field must have directed normally to the disc. The solid angle for the
dis at is given by
2 θ
ω = ∫ dφ ∫ sin θdθ = 2π (1 − cosθ )
0 0

 r 
∴ Fn = −γs .2π (1 − cos θ ) = −2πγs 1 − 

 a2 + r 2 
The potential may be found from the relation
 r
φ = − ∫ Fn dr = 2πγσ ∫ 1 − 
 dr
 a +r 
2 2

= −2πγσ [ a + r − r]
2 2

The potential vanishes as a → 0 , when is very large


   a2 πγσa 2
φ = −2πγσ r 1 +
 − r  ~ −
    2r 2 r
(6) Use the Gauss’ theorem to obtain the field and its potential of a cylindrical shell
and a thin infinite rod at a point at distance apart.
Answer:
From cylindrical symmetry, the field must be normal to the axis of the cylinder. Therefore,
the axial field will vanish and the field will be in the direction only. We surround the given
cylinder by another one of radius r with unit height. Consider the density of distribution is .
From Gauss’ theorem, one gets

∫∫ F .d S = −4πγM ,
S
M = ∫∫∫ ρ dv
V

∫∫ F .ndS = −4πγ ∫∫∫ ρdv


S V

Fn (2π r.1) = −4πγρ (1)


2γρ
∴ Fn = − ∴ φ = −2γρ log r + c
r
The constant is arbitrary. It may be considered as zero.
The potential function depends on as r ⇒ ∞ , we have log r ⇒ ∞ .

79
If the cylindrical shell has a thickness, and its required to obtain the field inside this shell,
one gets
Fn = 0

Fn = −2γm
(r 2
− R12 1 )
(R 2 − R1 r
2 2
)
If the cylinder is rigid, then R = 0 , and m is the mass of the cylindrical shell whose height is
(
given by πρ R22 − R12 )
Fn = −
(
2γ r 2 − R12 ) (
πρ R22 − R12 )
(
r R2 − R1
2 2
)
Fn may be written as

Fn = −
r
πρ r 2 − R12( )
(7) If the potential distribution as given as
 r cos θ ;r<a

φ (r ,θ ) =  a 3
 r 2 cos θ ;r > a

Find corresponding field and the surface density at the sphere r = a


Answer:
• As r < a , we have
∂φ 1 ∂φ
∇φ = er + eθ
∂r r ∂θ
= cos θ e r − sin θ eθ
 F = ∇φ ⇒ F = − cos θ e r + sin θ eθ
• As r > a , we have
2a 3 a3
∇φ = − cos θ e r − sin θ eθ
r3 r3
2a 3 a3
F = 3 cos θ e r + 3 sin θ eθ
r r
Therefore, the field distribution may be written as
 − cos θ e r + sin θ eθ r<a
 3
F = a
 r 3 (2 cos θ e r + sin θ eθ ) r>a

To find the surface density σ on the sphere r = a , we have


Fn 2 − Fn1 = −4πγσ
∂φ1 ∂φ 2
Or − = −4πγσ
∂r ∂r
3 coσ θ
σ =−
4πγ

80
(8) If the potential distribution as given as
 ka  1
(
 − 3 1 + 5a 2 2 z − x − y 
2 2 2 
; r<a )
  
φ ( x, y , z ) = 
 
( )
3
− ka  a + a 2 z 2 − x 2 − y 2  ; r>a
 3  r 5r
5

Find the volume density ρ and the surface density σ on the sphere r = a
Answer:
To find the volume density, from the potential Poisson’s equation, we have
∇ 2φ = 4πγρ
The surface density σ may be obtained as the previous example
The Laplacian operator in the spherical coordinates may be written as
1 ∂  2 ∂  1 ∂  ∂  1 ∂2
∇2 ≡  r  + 
sin θ  +
r 2 ∂r  ∂r  r 2 sin θ ∂θ  ∂θ  r 2 sin 2 θ ∂φ 2
r 2 = x2 + y2 + z2
2 z 2 − x 2 − y 2 = 3z 2 − ( x 2 + y 2 + z 2 )
= 3r 2 cos 2 θ − r 2 = r 2 (3 cos 2 θ − 1)
Therefore, we have
 ka  
 − 1 +
r2
(
3 cos 2 θ − 1  ) ; r<a
 3  5a
2
φ (r , θ ) =  
− ka  a + a 3 cos 2 θ − 1 
(( ))
3
; r<a
 3  r 5r 3 

Hence, one gets
∂φ1
∂r
ka  2r
3  5a
(

= −  2 3 cos 2 θ − 1  )

∂φ 2 ka  a 3a 3 
∂r
= − − 2 − 4 3 cos 2 θ − 1 
3  r
( )
4r 
∂φ1 ∂φ 2
− = −4πγσ at r = a
∂r ∂r
Therefore,
The volume density ρ is given from
1
ρ= ∇ 2φ
4πγ
• At
1 ka 1
ρ= .− . (4 − 2 − 2) = 0
4πγ 3 5a 2
ρ =0
• At

81
1  ka 1 ∂   ka ∂  
ρ=  . 
4πγ  3 r ∂r 
a +
3a 3
3 cos(2
θ − 1  + ) 1
sin θ
a3
(6 cos θ sin θ ) 
 3 r sin θ ∂θ 
2 2 2 3
5r 5r 
=0
Therefore, the mass is distributed only on the surface by σ .

(9) If the potential distribution as given as


2
 3 πγr (3 x − a ) ; r<a
2 2

φ ( x, y , z ) =  5
 2 πγr a (3 x 2 − y 2 − z 2 ) ; r > a
 3 r5
Find the volume density ρ and the surface density σ on the sphere r = a
Answer:
In this case, instead of the axis, we may consider the axis as the polar axis, in
which the angle θ is measured from it.
Therefore, we may write
z = r sin θ sin φ & y = r sin θ cos φ & x = r cos θ
Substitute for x, y, z in the given distribution
2
 3 πγr (3r cos θ − a ) ; r<a
2 2 2

φ (r ,θ ) =  5
 2 πγr a (3 cos 2 θ − 1) ; r>a
 3 r5
Hence,
∂φ1
= 4πγρ ρ cos 2 θ
∂ρ
∂φ 2
= −2πγρ 4 (3 cos 2 θ − 1)
a5
∂ρ ρ
a5
= −4πγρ 3 (cos θ sin θ )
ρ
∂φ1 ∂φ 2
σ = ρa (1 − 5 cos 2 θ )
1
At r = a − = −4πγσ
∂r ∂r 2
Substitute in the Poisson’s equation, one finds
r ; r<a
r=
0 ; r>a
This shows that the mass is distributed like the solid sphere with a uniform volume density
ρ and surface density σ .

82
Exercises

1. If the potential function φ of an attraction field is given by φ = e − r , where is the


3

position of any point measured from the origin, prove that the volume density is

given by r =
3r
4πγ
( )
3r 3 − 4 e − r .
3

2. If the potential distribution as given as


 − λ r cos θ ; r<a

φ (r ,θ ) =   2a 3 
λ cos θ  r − 2  ; r>a
  r 

Find the volume density ρ and the surface density σ on the sphere r = a . Also,
find the total mass on the sphere, and the components of the field at any point.
3. If the field F = r −5 ( A . r ) r + r −3 B is a conservative field, prove that A = −3B , and
the suitable potential is given by φ = (B . r ) r −3 , where A and B are constant
vectors.
4. If the Field distribution as given as
 0 ; x<0

F ( x, y , z ) =   r 1 r 2 
λ −  ; x>0
  r13 r23
  
where r 1 is the joining between the points (0,0,0) and (x, y, z ) and r 2 is the joining
between the points (− a,0,0 ) and ( x, y, z ) . Find the surface density on the plane
Also, find
(i) Prove that the volume density ρ vanishes on the space x > a and the surface
density on the plane x = 0 .
(ii) The potential distribution at any point in the space.
5. If the potential distribution φ at any point in the space ( x, y, z ) is given by
0 ; r<a
 c
f = − + ca
; r>a
 r1 f r2
where r1 is the distance from the points ( f ,0,0 ) and ( x, y, z ) and r2 is the distance from the
 a2 
points  ,0,0  and ( x, y, z ) . Find the field F at any point on the space. Hence, or
 f 
otherwise, Find the surface density on the surface r = a .

83
 a3 
6. If the potential at any point outside the sphere r = a is  − a cos θ (r − 2 )  , where
 r 
r is the distance from its center and θ is the confined angle measured from a
constant diagonal. Find the surface density on the sphere and prove that the volume
density is zero.
7. Consider a uniform distribution with an axial symmetry about the origin, and is the
distance from the origin to any point in space. If the total mass inside a sphere of
 r 2  − r / a 1 − 2 r / a 
radius is given by a 2  e − e  . Prove that the potential function is
 a  2 
 γ 1 
given by a  e − r / a − e − 2 r / a  . Also, show that as r → 0 , the volume density is
 a 2 
3a
given by .
4πa 3
8. Find the potential of a uniform ring at any point on its axis,
9. Find the potential of a uniform disc at any point on its axis,
10. Find the potential of a uniform spherical shell at any point.
11. Find the potential of a uniform solid sphere at any point,
12. Prove that the attraction field at any point is perpendicular to the equipotential
surfaces.

84
CHATER IV
Hydrodtatics
Fluid Statics

1- Fluid definition:

Fluids are substances which are capable of flowing and which


conform to the shape of containing vessels. When in equilibrium, fluids
cannot sustain tangential or shear forces. All fluids have some degree of
compressibility and offer little resistance to change of form.

Fluids may be devided into liquids and gases. The chief differences between
liquids and gases are (a) liquids are practically incompressible whereas gases are
compressible and often must be so treated and (b) liquids occupy definite volumes
and have free surfaces whereas a given mass of gas expands until it occupies all
portions of any containing vessel.

2- Pressure:
The pressure of the fluid at any point is the compression strength on a
unit areas on surface immersed at this point so that this surface does not
affect the arrangement of liquid particles.

Theorem:” The pressure at any point of a fluid at rest is the same for all directions”.
Proof:
Let a volume of the fluid draw as a tetrahedrom as shown in the figure.
Since the tetrahedrom is at rest , then the all forces on it= zero.
by resolving the forces in x- direction, we get

P2 dy dz − P3 sin αdz ds = 0 (1)


Since ds sin α = dy , then
P2 dy dz − P3 dy dz = 0
P2 = P3
By resolving in y- direction, we get
1
P1 ds dx − P3 dz ds cos α − gdx dz = 0
2
Since ds cos α = dx , then
1
P1 − P3 − g dy = 0
2

If the volume of the tetrahedrom tends to zero ,then dy is also tends to zero, then
P1 = P3

85
which prove the thorem .

3- Equilibrium equation:
Suppose that balanced fluid under the influence of external forces
X for unit mass in the fluid.

To find the equilibrium equation for any point P draw a surface ∆S bounded by the
volume around ∆V around the point, then the element have balanced the impact of
two forces.

(1) The external forces F e that results from the force x for unit volume in the
form + ∫∫∫ X dV .
∆v

(2) The internal forces F b that results form the liquid pressure on the volume
are − ∫∫ P . d S .
∆s

From the equilibrium, we get

Fe + Fb = 0 (1)
 
∫∫∫ ρ X dv +  − ∫∫ Pd S  = 0
∆v ∆s

From green’s theorem

∫∫ ∇ P dV = ∫∫ P . d S
∆v ∆s

then

∫∫∫ ρ X dS − ∫∫∫ ∇ P dV = 0
∆v ∆v

∫∫∫ (b X − ∇P ) dV = 0
∆v

But V is arbitrary volume, then


ρ X = ∇P (2)

From (2) we get that the force of fluid pressure on the unit volume is − ∇P

The equilibrium of fluid under the influence of gravity.

86
Prove that the pressure on the unit area at any point is equal to the weight
of the liquid column above this point.

Proof:
Suppose that ρ is the density balanced under the influence of gravity , then the
external forces for unit volume is

ρ X = −ρ g k
but ρ X = ∇P
Then the component of ∇P is in the direction of z only,i.e.

∂P
− ρg = −
∂z
P = − ∫ ρ g dz (1)

By integration of (1) if ρ constant ( the liquid is incompressible)


P = −ρ g z + c

If P0 is the atmospheric pressure, then P = P0 at z = z 0 and P0 = c − b g z ,i.e.


P − P0 = − ρ g ( z − z 0 ) (2)

4-Pressure on plane submerged bodies

If an lamina surface S is flooded in liquid, then the liquid press on the body by
normal pressure force. To calculate the pressure force let an element dS
in the body and its depth for free surface is z, then the liquid pressure at this
element is ρ g dS and in the normal direction of the level of the lamina , then the all
pressure is

F = ∫∫ ρ g z dz = g ∫∫ ρ z dS
S S

If Z is the depth of the center of gravity then

∫∫ ρ ' z dS ∫∫ ρ ' z dS
Z= S
= S

∫∫ ρ ' dS ρ'S
S
where ρ ' is the density area element for the body

87
∫∫ ρ ' z dS = Z ρ ' S ⇒ ∫∫ ρ z dS = Z ρ S

If z f is the depth of the center of pressure, we take the moment around the axis X,
we get
The total moment of the forces = The moment of the resultant.

∫∫ ρ ε z dS = Z f ∫∫ ρ g z dS
2

∫∫ ρ gz dS = ∫∫ ρ z dS = ∫∫ ρ ' z dS
2 2 2
I xx
Zf = =
∫∫ ρ g z dS ∫∫ ρ g z dS ∫∫ ρ ' z dS Sρ ' z

If we want to find the distance between the center of pressure and the axis Z,
we take the moment around the axis (i.e. Z f ), then

the moment of inertia around x axis


depth of the center of gravity × the area × the surface density

∫∫ ρ g z dS = X ∫∫ ρ g z dS
f

x =
∫∫ ρ ' z x dS = I xz

∫∫ ρ ' z dS ρ ' S z
f

K xx2
=
z
Example (1):
Find the center of pressure on the rectangular lamina immersed in homogeneous
liquid if one of the corners of the lamina is in the liquid surface.

Solution:
Let the rectangular ABCD be immersed in the liquid with AB in the free surface ,
AC=a and AB =b. Take an small element HK from the area of the lamina at x from
AB and its width dx.
The pressure of the element = dF

dF = (d dx)( x g ρ )
= bρ x dx g

We notice that the pressure acts at the middle of the element HK and normal on the
plane of the lamina,i.e. the resultant pressure F acts at a point on EG .

We can get resultant pressure F as,

88
a a
1 2 1 2
F = ∫ bρxdx = bρ x = a bρ
0
2 0 2
By taking the moment around AB , we get

a
F X f = ∫ (b ρ xdx) x
0
a
1 2 1 1
a bρX f = bρ x 3 = a 3 bρ
2 3 0 3
2
Xf = a
3
Example (2): Solve the previous example if the lamina is inclined to the plane with
angle α .

Solution:

dF = (b dx)( x sin α ρ )
α
1 2
F = ∫ b ρ sin α x dx = α ρb sin α
0
2
By taking the moment around AB , we get

1
F X f = ∫ (b ρ x sin α dx) x = α 3 b ρ sin α
3
2
Xf = a
3
If θ = 0 , i.e. the lamina is horizontal, the center of the pressure coincide at the center
of the gravity of the lamina.

Example (3):
A triangle is immersed in a liquid with the base horizontal and vertex in the surface,
find the center of pressure if the plane is inclined by angle α with the horizontal.

Solution:
Let the triangle is ABC , its vertex is in the free surface and the side BC is horizontal.
Let α be the inclination of the plane area to the horizontal. We take small element
HK where HK parallel BC ,let x is the distance between the element and the
intersection of the lamina plane and the free surface, and the element width is dx.

89
( )
dF = HK dx ( x sin a ) ρ
HK x ax
= → HJK =
a a e
but
α 2
x sin αρ dx
dF =
e
1
∴ F = ∫ dF =  2 α sin α
3
By taking the moment around DE , we get

FX f = ∫ x dF
1 2  sin α 2 
 αρ sin α X f = ∫  α ρ x dx x
3 0
  
1 sin α 4
= αρ 
4 
i.e. the center of the pressure 0 lie at the middle straight line AG and the distance
3
from DE is .
4

Example (4):
A triangle area immersed in a liquid with its base in the surface and its plane is
inclined with the horizontal by α , find the center of the pressure.
Solution:
We take a small element HK parallel to BC at distance x from BC and its thick dx .
Let the hight A equal

HK  − x
⇒ HK = ( − a ) . 
a
=
a  
dF = ( HK dx) (the depth of the element from the surface) ×ρ
dF = ( HK dx) x sin α ρ

= sin α x( − x) dx

The center of the pressure is in the middle of the element.

a ρ sin a

F = ∫ dF = ∫0 x ( x − ) dx

1
α ρ  sin α
F=
2

6
By taking the moment around BC , we get

90
F X f = ∫ x dF
1
Xf = 
2

i.e. the center of pressure is at the middle of AD

Example (5):

A circular disk of radius a is immersed vertically in liquid such that the center of the
disk is in depth  under the free surface of the liquid.

Solution:

We take an element from the area of the disk as a horizontal strip HK at distance x
from the horizontal diameter CB and its depth dx. The pressure on this element is
dF.

dF= the area of the element×the depth of the gravity× the density of the liquid

(
= 2 a 2 − x 2 dx ( − x )ρ )
The total pressure F
F = πa 2 ρ

By taking the moment around CB , we get

Fx f =
∫ x df
+a
=2
∫ −a
( − x ) a 2 − x 2 x dx
By integraion

+a
I=
∫ −a
( x − x 2 ) z 2 − x 2 dx

+a +a
=
∫ −a
x a 2 − x 2 dx −
∫ −a
x 2 a 2 − x 2 dx

The first part of the integration is odd, i.e. f (−a ) = −f (a )


Then the result is zero, but the second part we use the substitution

91
x = a sin θ
dx = a sin θ dθ
π
I = −2∫ 2 (a 2 sin 2 θ )(a cosθ )(a cosθ dθ )
0
π
= −2a ∫ sin 2 θ cos 2 θ dθ
4 2
0
π
= −2a ∫ sin 2 2θ dθ
4 2
0

I = − a 4 ∫ [1 − cos 4θ ]dθ
1
4
π
1  1  2
= − a 4 θ − sin 4θ 
4  4  0

1 π  π
= − a4  = a4
4 2 8
a2
∴xf = −
4

If  = a, then
1
xf = a,
4

under the center of the disk.

Exercises

(1) The width of the rectangular door is 5F and the hight of the water
on one of its side is 12F, and the other side is 7F.Find the pressure
at the door and obtain the influence point.

(2) ABCD is rectangular lamina immersed such that the horizontal length is
the upper and DA, BC is vertically and their lengths is , and the middle
of AB is E. Find the depth of AB from the free surface such that the total
pressure from the distances which divide the rectangular by the planes
EC,DE are in the ratio 1 : 1 : 3

92
Chapter V
Moment of Inertia

First:
We studied before that the name of the moment of a mass with respect to any axis
r × m is the first moment, but the second moment ( the moment of inertia) with
respect to the axis passing through the point 0 is ∑ mr 2 . We can say that
∑ mr 2
= MK 2 , where M is the mass of the body and K is the radius of the inertia
with respect to the axis.

Second:
If we suppose that the elemnt m at the position ( x, y ) with respect to the two
perpendicular axis , then ∑ mxy is the product of inertia with respect to 0 x & 0 y

The principal facts:

(1) The moment of inertia of a body with respect to any axis = The total of inertia
for its parts with respect to the same axis . This result is true in the case of the
product of inertia.

(2) The normal axis theorem:

The sum of moment of inertia of an area with respect to the normal axis lie in
the plane of the area is equal to the moment of inertia of the area with respect
to any axis passing through the point O and normal to the plane.

∴ the moment of inertia with respect to 0 x

I 0 x = ∑ ( y 2 .∆M )
and

I 0 y = ∑ ( x 2 .∆M )
∴ I 0 x + I 0 y = ∑ ( x 2 + y 2 ).∆M = ∑ r 2 .∆M
(3) Parallel –axis theorem:

If the moment of inertia for a body of mass M with respect to any axis passing
through the center (O).
∴ the moment of inertia with respect to any axis parallel to the first axis and
the distance between them is a = I + Ma 2

93
Special cases:

(1) The moment of inertia of a bar has length is 2L.

with respect to an axis perpendicular at any end of a bar.

Proof:
consider the mass of the bar AB is M , let an element from the bar with length ∆x
at distance x from the end A , then ∴ ∆M = ∆x.ρ ,where ρ is the density of the bar.

∴ M = 2L.ρ (1)
∆I A = x .∆m = ρx ∆x
2 2

2L 8 8 L3 M
∴IA = ρ∫ x 2 dx = L3 ρ = ×
0 3 3 2L
4
∴IA = ML
3

(2) ) The moment of inertia of a bar has length is 2L. with respect to an axis
perpendicular at the middle of the bar and we consider the bar is consist of
M
two parts the length of each is L and each mass is .
2

The moment of inertia of the bar = the sum of the moment of inertia for its parts.

2
4  M  L 
I 0 = 2 ×   
3  2  2 
1
I 0 = ML2
3

(3) The moment of inertia of a bar has length is 2L. with respect to an inclined axis
with angle θ .

∆ I 0 L = ∆m l 2 , l = x sin θ

∆I oL = ρ ( x sin θ ) ∆x = ρ x 2 sin 2 θ ∆x
2

a a
x3
I 0L = ρ sin θ ∫ x dx = ρ sin θ
2 2 2

−a
3 −a

94
2
I 0L = ρ a 3 sin 2 θ
3
M
 M = 2a ρ ⇒ ρ =
2a
1
I 0L = Ma 2 sin 2 θ
3

(4) The moment of inertia of a rectangular of lengths (2a,2b) with respect to an axis:
(a) coincide with the length 2b.
dividing the rectangular into strips parallel to 2a ,i.e. normal to 2b.

4 2
∴ ∆I oy = a × ∆M
3
4
∴ I oy = Ma 2 (1)
3

(b) coincide with the length 2a

4
I ox = Mb 2 (2)
3
(c) with respect to an axis passing through the center and parallel to 2b.

1
INy = Ma 2 (3)
3

(d) with respect to an axis passing through the center and parallel to 2a.

1
INx = Mb 2 (4)
3

(e) with respect to an axis normal to its plane and passing through the center.
1
I N = M (a 2 + b 2 )
3
(f) The moment of inertia of a circular ring of radius "a".

first: with respect to the axis perpendicular to its plane and passing
through its center.

we choose an element ∆M
then
∴ ∆I C = ∆M × a 2 by integration,
∴ I C = Ma 2

95
second: with respect to any diameter.

By using parallel –axis the


2 I = I C = Ma 2
M 2
∴I = a
2

(6) ) The moment of inertia of a circular disk has radius a with respect to a
perpendicular axis on its plane and passing through the center.

An annular element of circular area , its radius is x and its thickness is ∆x .

∴ ∆M = 2πx.∆x.ρ , where

M = πa 2 ρ
∴ ∆I C = ∆M .x 2 = 2πx 3 ∆x.ρ

By integration

a π
I C = ∫ 2πρx 3 dx = a 4 .ρ
0 2
M π a2 × M
ρ = ∴ IC =
πa 2 2 πa 2
M 2
IC = a
2

Remark:
The moment of inertia of a circular disk has radius a with respect to the
M 2
diameter is a . (by using the normal axis theorem):
4
1 Ma 2
2I = I C → I = I C =
2 4
Results:
First: The moment of inertia of a hollow cylinder with respect to its axis= Ma 2 .
Ma 2
Second: The moment of inertia of a solid cylinder with respect to its axis = .
2

(7) The moment of inertia of a solid cone with respect to its axis

96
Suppose that the mass of the cone M ,its base radius a and its height h .
Consider an element as a disk , its radius is x , its thickness ∆y and its mass is ∆M .

∆M = π x 2 ∆y ρ , where ρ is the unit volume mass from the cone

The moment of inertia of the element with respect to normal axis passing
through the center.

1 1
∆I oy = ∆M x 2 = πρ x 4 ∆y
2 2

The moment of inertia of the cone with respect to its axis


h
1
I oy = πρ ∫ x 4 dy (1)
2 0
From the similarity we find relation between x & y

⇒ x = (h − y )
x a a
= (2)
h− y h h

from (1),(2) we get

a 4πρ a 4πρ
[ ] a 4πρh
h

4 ∫
( ) (h − y )5 (3)
h
I oy = − = − =
4
h y dy 4 0
2h 0 10h 10
but the mass of the cone M is

1 3M
M = πa 2 h ρ ⇒ ρ= (4 )
3 πa 2 h
from (3),(4) we get

3
I0y = Ma 2
10

(8) The moment of inertia of a triangle with respect to one of its ribs:

Let the strip b1c1 // bc


b1c1 h − x
=
bc h
h−x
∴ b1c1 = × bc
h
h−x
∴ ∆M = b1c1 .∆x.ρ = .bc.∆x.ρ
h

97
h−x
∴ I BC = ∆M .x 2 = bc.x 2 ∆x.ρ
h
h
bc.ρ h bc.ρ  x3 x4 
∴ I BC = ∫0 (h − x) x dx = h h − 
2

h  3 4 0
bc.ρ  h 4 h 4 
=  −  (1)
h  3 12 
1 2M
but M = bc.h.ρ → ρ= (2)
2 bc.h
From (2) in (1)
bc 2 M h 4 Mh 2
I BC = × × =
h bc.h 12 6

Result:

The moment of inertia of a triangle its mass is M with respect to one of its ribs
M
equal to the moment of inertia of three masses them all equal and in the middle
3
of the ribs.

(7) The moment of inertia of a hollow sphere with respect to its diameter.

Consider an element as a ring has a radius y and mass ∆M , such that


∆M = 2π ρ a∆θ ρ where ρ is the density per unit volume

The moment of inertia of the element about the diameter AB is


∆ I AB = ∆M y 2 & y = a sin θ
∆ I AB = 2π y 3 a∆θ ρ
π π
I AB = 2π a 4 ρ ∫ sin 3 θ dθ = 2π a 4 ρ ∫ sin 2 θ sin θ dθ
0 0
π
( )
I AB = 2π a 4 ρ ∫ 1 − cos 2 θ sin θ dθ
0

π π

= 2π a 4 ρ  ∫ sin θ dθ − ∫ cos 2 θ sin θ dθ 
0 0 
π
 π 1 
= 2π a 4 ρ cos θ 0
+ cos 3 θ 
 3 0 

 1
( 
= 2π a 4 ρ − (−1 − 1) + (−1) 3 − (1) 3  )
 3 
 2 8
= 2π a 4 ρ 2 −  = πa 4 ρ
 3 3

98
but
M
M = 4π a 2 ρ ⇒ ρ=
4πa 2
2
∴ I AB = M a2
3

(8) ) The moment of inertia of a solid shere with respect to an axis passing through

its center:

Let an element like to a hollow sphere with radius r and thickness ∆r it has the
mass ∆M ,
, where ρ is the density per unit volume.

∆M = 4π r 2 r ∆r

The moment of inertia of the element ( hollow shere)

2
∆I 0 x = ∆M r 2
3
8
= π r r 4 ∆1
3
then

a
8 8
I 0 x = π r ∫ r 4 dr = π r a 5
3 0
15

but

4 3M
M = π a3 ρ ⇒ ρ=
3 4πa 3
2
∴ I 0x = M a2
5
(9) The moment of inertia of an ellipse which axis are 2a & 2b

The equation of the ellipse is


x2 y2
+ =1 (1)
a2 b2

99
The parametric equations for the ellipse are x = a cos θ , y = b sin θ , let the
strip element has a length 2 y , its thickness is ∆x and its mass is ∆M

∴ ∆M = 2 y∆x.ρ
= 2b sin θ .(−a sin θ ).ρ .∆θ
= −2ba sin 2 θ .ρ∆θ
Where M = πab.ρ (2)
∆M 2 2
∴ ∆I ox = y = − ab 3 sin 4 θ .∆θ .ρ
3 3
π
2
4
I ox = ab 3 ρ ∫ sin 4 θ dθ
3 0

4 3 3 1 π πab 3 ρ
= ab ρ × × × = (3)
3 4 2 2 4
then

π M M
∴ I ox = ab 3 × = b2
4 πba 4
M 2
∴ I oy = a
4

By using the normal axis theorem


M 2
Io = (a + b 2 )
4

Example (1):

Find the moment of inertia of a bar of length 2 with respect to a normal axis
passing through its middle point if ρ(x) is the unit length mass which prprtional to
the distance from the middle point by the relation

ρ ( x) = a + b x
Solution: consider an element ∆x at distance x from the middle axis
then the moment of inertia of the element is

∆I = ρ ( x).∆x.x 2
( )
= a + b x x 2 ∆x

100
+
( )
∴ I = ∫ a + b x x 2 dx
−

2
I = a 3
3

Example (2)

ABCD is a rectangular lamina with mass M, BC = 6L , AB = 4L and its center is O.


If we isolate a disk with center O and radius L from the lamina find
the moment of inertia with respect to
1- AB, 2- BC, 3- The point B.

Solution:
M × πL2 πM
The mass of the disk = = .
24 L2 24
the moment of inertia with respect to AB = the moment of inertia of the rectangular
with respect to AB - the moment of inertia of the disk with respect to AB

4 M 
I AB = M (3L) 2 −  1 L2 + M 1 (3L) 2 
3  4 
2
4 M L
= M × 9 L2 − 1 × 37 (1)
3 4
πM ML2
= 12 ML2 − × 37 L2 = [1152 − 37π ]
96 96

4 M 
I BC = M (2 L) 2 −  1 L2 + M 1 (2 L) 2 
3  4 
16ML2 17 M 1 L2 16 17 πM 2
= − = ML2 − × L
3 4 3 4 24
ML2
= [512 − 17π ] (2)
96
To find the moment of inertia with respect to the point B we use normal axis
theorem.

ML2
I B = I AB + I BC = [1664 − 54π ]
96

Example (3)

Find the moment of inertia of a circular ring with mass M and radius a with respect
to an axis passing through a point on its circumference and normal to its plane.

101
Solution:
Find the moment of inertia of a circular ring with mass M and radius a with respect
to a normal axis on its plane is

I 0 = Ma 2 (1)

By using parallel axis theorem, the moment of inertia of a circular ring to an axis
passing through a point on its circumference and normal to its plane is

I A = I 0 + Ma 2 (2)

from (1), (2)

I A = Ma 2 + Ma 2
I A = 2Ma 2
Example (4):

Find the moment of inertia of a half circular lamina with radius a with respect to a
tangent parallel to its diameter.

Solution:
Let the mass of the lamina M, then
1
I bb ' = Ma 2
4
If we take c is the center of gravity of the lamina, then
2
I bb ' = I cf + M .oc
and
2
I ne = I cf + M .cn
By subtracting we get

(
I ne = I bb ' = M cn − oc
2 2
)
then
I ne =
1
4
Ma 2 + Ma cn − oc ( )
1 5 8 
I ne = Ma 2  − 
4  8 3π 

Example (5):

102
ABCD is a rectangular lamina of lengths 2a,2b and mass M if we isolate an ellipse
with axis 2a, 2b from it and its center coincide with the center of the rectangular o.
find
1- the moment of inertia around the center o.
2- the moment of inertia around BC.

Solution:

4ab : πab
πabM πM
the mass of the ellipse M1 = =
4ab 4
M 2 M
I0 = I0 − I0 = (a + b 2 ) − 1 (a 2 + b 2 )
3 4
(a + b )
2 2
a2 + b2  3πM 
= [4 M − 3M 1 ] = 4 M −
12 12  4 
(a 2 + b 2 ) M
= [16 − 3π ]
48

Exercises

(1) A circular disk with radius a, has center 0 and mass M, find
- the moment of inertia of the disk with respect to a tangent of the disk and lie in
its plane.
- the moment of inertia of the disk with respect to a normal axis on the disk plane
and passing through any point on the circumference.

(2) Solve the latest exercise in the case of a ring.

(3) ABC is a right angle triangle lamina in B which AB = 8L , BC = 6L and if we


isolated a circular disk tangential to the triangle lips, find
- the moment of inertia of the remaining part around AB, BC.
- the moment of inertia of the remaining part around B.

(4) Determine the moment of inertia of a right circular cone with respect to
- an axis through the apex of the cone and perpendicular to its longitudinal axis.
- an axis through the centroid of the cone and perpendicular to its longitudinal
axis.

(5) Find the moment of inertia of a plate on the shape of the ellipse 16 x 2 + 9 y 2 16
with respect to an axis perpendicular to the plane of the plate and passing
through its center.

(6) ) Find the moment of inertia of a circular ring with mass M and radius a with
respect to an axis tangential to any points on it and parallel to its diameter.

103

You might also like